You are on page 1of 69

Question 1

Regarding amputation of foot


There is a difference in stance phase and propulsion when amputation of the toes is performed for
ischemia compared to trauma
Wound healing is not a problem with central ray amputations.
In Transmetatarsal amputations coverage obtained by Long plantar flap is no better than fish-mouth
planto dorsal flap
Hind foot amputation requires tibialis posterior transfer to prevent equinus deformity
Gait pattern and load transfer are severely affected following Symes amputation.

References
MILLER (CHAPTER FOOT AND ANKLE)

Question 2
Which is the most avascular disc?
L1/L2
L2/L3
L3/L4
L4/L5
L5/S1

References
Basic
Orthopaedic
First
Edited
Published
page 135, Chapter 15

Sciences
by
by-Hodder

The

Stanmore

Manoj
Arnold,

Question 3
An excision arthroplasty for an infected shoulder replacement results in:
Pain relief in all patients
High patient satisfaction
Similar function as two stage arthroplasty
Poor functional outcome
High rate of recurrent infection

References

guide,
Edition
Ramachandran
London

D. M. Rispoli, J. W. Sperling, G. S. Athwal, C. D. Schleck, III, and R. H. Cofield


Pain relief and functional results after resection arthroplasty of the shoulder
J
Bone
Joint
Surg
Br,
Sep
2007;
89-B:
1184
1187.
Despite applying this procedure principally to failed shoulder replacements, the results were similar to
those reported in the literature for patients after severe fracture-dislocation. Reduction of pain is
possible in one half to two-thirds of patients. The outcome of this operation in providing relief from
pain cannot be guaranteed, but the shoulder is usually comfortable at rest, albeit with profound
functional limitations.

Question 4
The following statements are true about metacarpals:
The two radial metacarpals form the fixed, stable longitudinal axis.
The 4th and 5th are more mobile than 1st and 2nd metacarpal.
The dorsal surface is the tensile surface, while the volar is the compressive surface.
All of the above.
None of the above

References
Rockwood and Green

Question 5
Which of the following nerves does NOT innervate any of the rotator cuff muscles?
Axillary nerve
Dorsal scapular nerve
Upper subscapular nerve
Lower subscapular nerve
Suprascapular nerve

References
Miller
MD,
Review
of
Orthopaedics,
4th
edition,
pg602
Dorsal scapular nerve supplies the rhomboids and levator scapulae which are not part of the rotator
cuff.

Question 6
During a plantar fascia and nerve release as described by Schon and Baxter which nerve or its
branch is released?
Sural nerve
Lateral plantar nerve
Superficial peroneal nerve

Medial plantar nerve


Deep peroneal nerve

References
Campbell's
pp1916-1919

Operative

Orthopaedics,

volume

Two

Question 7
Trapezius is supplied by which of the following nerves?
Suprascapular
Thoracodorsal
Dorsal Scapular
Spinal accessory
Axillary

References
Trapezius is supplied by the spinal root of the accessory nerve (CNXI). Thoracodorsal nerve supplies
latissimus dorsi. Dorsal Scapula nerve supplies the Rhomboids and levator scapulae. Suprascapular
nerve supplies Supraspinatus and Infraspinatus. Axillary nerve supplies Deltoid and Teres minor.

Question 8
Regarding diabetic foot
ABPI of > 0.6 indicates good wound healing potential
Transcutaneous oxygen tension of < 40mmHg corresponds to greater likelihood of wound healing
Total lymphocyte count and albumin are not predictors of wound healing
Peripheral neuropathy is assessed by metallic filament (Semmes-Weinstein)of varying thickness.
Treatment of Wagner grade-3 diabetic foot is nonsurgical

References
Question 9
A 35 year old builder frequently using vibrating tools, predominantly complains of
paraesthesias in the right little and ring fingers with no motor loss. The likely site of
compression neuropathy is
Bicipital aponeurosis
Arcade of Frohse
Guyon's Canal
Carpal tunnel

Pronator syndrome

References
Question 10
The Arcade of Struther's is a potential site for the compression neuropathy of which of the
following nerves
Median nerve
Ulnar nerve
Anterior Interosseous nerve
Posterior Interosseous nerve
Radial nerve

References
Green
Textbook
of
operative
hand
surgery
Arcade
of
Struthers
In middle of the arm, ulnar nerve pierces medial intermuscular septum and descends in front of the
medial head of the triceps. In 70-80% of individuals, nerve passes under arcade of Struthers. The
arcade is a thin aponeurotic band extending from medial head of triceps to the medial intermuscular
septum. It lies approximately 8 cm proximal to the medial epicondyle. The arcade may appear as
medial triceps muscle fibers crossing superficial to the ulnar nerve. The arcade is not site for
entrapment under ordinary circumstances, but it may become point of kinking if anterior transposition
of ulnar nerve is performed. Following transposition procedures, the surgeon should release the
arcade if it appears that the nerve is under tension. The ligament of Struthers (different from arcade
of Struthers) courses between the supracondylar process and medial epicondyle and can couse
median nerve compression.

Question 11
Which of the following is NOT a recognised complication of hamstring lengthening?
Popliteal Nerve Palsy
Femoral Nerve Palsy
Sciatic Nerve Palsy
Knee Recurvatum
Worsened Lumbar Lordosis

References
Campbell's
operative
Wheeless' textbook of orthopaedics.

orthopaedics,

10th

Question 12
Which of the following statements regarding radiological investigation of the hip is FALSE?

ed.

Bone marrow in the normal femoral neck has high signal intensity on T1 weighted image and low
signal intensity on T2 weighted image
MRI is more accurate than radionuclide scanning for the detection of occult femoral neck fractures
MRI is more sensitive than radionuclide scanning for the diagnosis of osteonecrosis of the femoral
head
The "figure of 8" or "cross-over" sign on plain X-ray may be a feature of Cam-type Femoroacetabular
Impingement
MRI within 48hrs of femoral neck fracture has not been shown to be useful for the assessment of
femoral head viability

References
OKU 8, Oxford Textbook of Orthopaedics

Question 13
Which of the following is NOT a method of reducing a native dislocated hip?
Rochester Method
Bigelow's Manoeuvre
Allis's Manoeuvre
Barlow's Manoeuvre
Gravity Method of Stimson

References
Reduction
of
posterior
dislocation
of
the
hip
in
the
prone
position.
Herwig-Kempers A. Veraart BE. Journal of Bone & Joint Surgery - British Volume. 75(2):328, 1993
Mar.
The East Baltimore Lift: A simple and effective method for reduction of posterior hip dislocation.
SJ Schafer and JO Anglen. Journal of Orthopaedic Trauma. Vol 13. No 1. 1999. p 56-57.
Posterior
HD

Hip
Skoff.

A Flexion Adduction
JL
Bassi
et

Dislocation,
Orthop
Method
al.

for

New
Rev.

the Reduction
JBJS.
Vol

Technique
1986.
of

for
p

Posterior Dislocatioon of
74-B.
1992.
p

Reduction.
101-105.
the Hip.
157-158

In the Rochester Method the patient is placed supine with uninjured hip and knee flexed (this knee
acts as a pivot point for the surgeon's hand). One of the surgeon's hands is place underneath the
injured knee and over top of the uninjured knee, this manuever flexes both the patient's injured hip
and knee. The other hand grab's the ankle (injured side) and this can be used to generate traction (by
pressing down on the ankle) and at the same time the ankle can be used to control hip rotation.
Reduction is obtained by traction, internal rotation, and then external rotation once the femoral hip
clears
the
acetabular
rim.
In Bigelows method patient lies supine, & assistant applies countertraction by downward pressure on
the ASIS. Surgeon grasps affected limb at ankle with one hand, places opposite forearm behind the
knee, and applies longitudinal traction in line of deformity. Adducted & internally rotated thigh is flexed
> 90 deg on abdomen. This relaxes the Y ligament and allows the surgeon to bring the femoral head
near the posteroinferior rim of the acetabulum - while traction is maintained, femoral head is levered

into
acetabulum
by
abduction,
external
rotation,
and
extension
of
hip.
In Allis method the patient is placed in the supine position. Knee is flexed to relax the hamstrings and
assistant stabilizes the pelvis and applies a lateral traction force to the inside of the thigh. Longitudinal
traction is applied in line with axis of femur, and the hip is slightly flexed. Essential feature is traction in
direct line of deformity, followed by gentle flexion of the hip to 90 degrees. Surgeon gently adducts &
internal rotates the femur to get reduction, hip is gently rotated internally & externally w/ continued
longitudinal
traction
until
reduction
is
achieved.
Stimsons method is described primarily for acute posterior dislocations, but anterior dislocations can
occassionally be reduced by this method. Patientt is in prone position with lower limbs hanging from
end of table. Assistant immobilizes the pelvis by applying pressure on the sacrum, hold knee and
ankle flexed to 90 deg & apply downward pressure to leg just distal to the knee. Gentle rotatory
motion
of
the
limb
may
assist
in
reduction.
Barlows test is a test for hip dislocatability. Following a positive Barlows test, Ortolanis test is used to
check the now dislocated hip can be reduced

Question 14
The mechanism of action of an antibiotic that may cause tendo Achilles rupture is:
INHIBITING DNA GYRASE
INHIBITING 70S & 80S RIBOSOMES
INHIBITING INSERTION OF GLYCAN SUBUNIT IN TO CELL WALL
INHIBITING PEPTIDOGLYCAN SYNTHESIS
INHIBITS RNA SYNTHESIS

References
MILLER,
FOURTH
EDITION,
Quinolones may cause tendon rupture in particular tendoachilles

PAGE

110

Question 15
Which of the following gait patterns is associated with hamstring shortening in a patient with
cerebral palsy
Circumduction Gait
Crouch Gait
Hip Hiking Gait
Shuffling Gait
Vaulting Gait

References
Campbells
operative
orthopaedics,
10th
ed.
Gait
analysis,
and
introduction,
M.W.Whittle
While other gait patterns can be related on CP, only the one is caused by hamstring shortening.

Question 16

Aging is associated with changes in the collagen fiber diameters and fibril concentrations.
Changes noted in aging are
Decrease in collagen fiber diameter and increase in collagen fibril concentration
Increase in collagen fiber diameter and collagen fibril concentration
Decrease in collagen fiber diameter and collagen fibril concentration
Increase in collagen fiber diameter and decrease in collagen fibril concentration
No change in collagen fiber diameter and collagen fibril concentration

References
Sargon
MF,
Doral
MN,
Age related changes in human PCLs: A light and
Knee surgery Sports Traumatol Arthrosc 2004;12:280-284

Atay
electron icroscopic

OA
study.

Question 17
Surgical treatment of tennis elbow commonly involves release of which tendon?
Extensor Carpi Radialis Brevis
Extensor Carpi Radialis Longus
Supinator
Extensor Carpi Ulnaris
Brachialis

References
Millers textbook of Orthopaedics

Question 18
In Total Hip Arthroplasty dislocation is a recognised complication. What is the safe range of
acetabular component anteversion?
0 +/- 5 deg
15 +/- 10 deg
30 deg
30 +/- 10 deg
45 +/- 10 deg

References
Lewinnek
Bidderman

GE
R

et
et

al,
al,

JBJS-Am,
JBJS-Br,

This position is thought to reduce risks of postoperative dislocation

1978;60(2):217-220
2005;87(6):762-769

Question 19
Slipped upper femoral epiphysis is associated with:
Hyperthyroidism
Renal osteodystrophy
Low weight
Maternal age
Paternal age

References
Puri-R. Smith-C-S. Malhotra-D. Williams-A-J. Owen-R. Harris-F. J-Bone-Joint-Surg-[Br]. 1985 Jan.
67(1).
P
14-20.
Miller,
MD;
Hart,
JA:
Review
of
orthopaedics,
5th
Edition,
Saunders
2008.
SUFE has been associated with hypothyroidism, renal osteodystrophy and obesity. Maternal aging is
associated with Downs syndrome while paternal aging is associated with achondorplasisa.

Question 20
Which of the following statements is correct with regards to knee bio-mechanics
Flexion and Extension occur about a fixed transverse axis of rotation
As extension progresses, shorter, more highly curved lat condyle exhausts its articular surface & is
checked by PCL
When Flexion is initiated, unscrewing of the joint occurs by external rotation of the tibia on the femur.
There is more than twice the weight-bearing stresses on the medial femoral condyle compared to the
lateral femoral condyle
Knee joint surface loads are upto four times body weight on climbing stairs

References
J
Bone
Joint
Surg
Br.
2004
Aug;86(6):925-31
Does the femur roll-back with flexion?Pinskerova V, Johal P, Nakagawa S, Sosna A, Williams A,
Gedroyc W, Freeman MA.

Question 21
Which of the following option is TRUE of the gluteus medius muscle:
Has an L5 nerve root innervation
Inserts onto the lesser trochanter of the femur.
Is an adductor of the hip joint.
Is an internal rotator of the hip joint.
Is innervated by the inferior gluteal nerve.

References
Grays

anatomy

The gluteus medius originates from the ilum and inserts into the greater trochanter. It is supplied by
the superior gluteal nerve with a predominantly L5 nerve root in addition to S1. Its action is to abduct
the hip.

Question 22
Which of the following options represents a grade 2 spondylolisthesis?
10% anterior slip of L5 on S1
20% anterior slip of L5 on S1
45% anterior slip of L5 on S1
55% anterior slip of L5 on S1
70% anterior slip of L5 on S1

References
Orthoteers
Grade
1
Grade
2
Grade
3
Grade
4
Grade 5 = >100% = spondyloptosis

=
=
=
=

<25%
25-50%
50-75%
75-100%

Question 23
Which one of the following statements regarding posterior cruciate ligament substituting TKR
is correct?
Sacrificing the PCL opens the extension gap more than the flexion gap
PCL resection effects lateral soft tissue balance
Over resection of the distal femur may result in a flexion contracture
Cam-post impingement may be caused by excessive femoral implant extension
The surgeon should aim for a tibial cut of maximal posterior slope

References
Surgical technique differences between posterior-substituting and cruciate-retaining total knee
arthroplasty.
Sierra RJ, Berry DJ. J Arthroplasty. 2008 Oct;23(7 Suppl):20-3. Epub 2008 Aug 12.
PCL sacrifice increases the flexion gap by between 1.8 and 4.8mm. To maintain equal flexion and
extension gaps more distal femur is resected in PCL substituting TKRs. Over resection of the distal
femur will cause a larger extension gap than flexion gap. Most PCL substituting TKR designs have a
cam and post mechanism to drive femoral rollback in flexion. An important drawback of this
mechanism is cam post impingement in extension when the anterior portion of the post contacts the

box. To prevent this the femoral cut should not be cut with excessive slope. In most PCL substituting
TKR designs the tibia should be cut with minimal posterior slope as removal of the PCL increases the
flexion space which will be increased further by increased posterior slope.

Question 24
What is the most common organism causing chronic deep joint infection following primary
total hip replacement?
Coagulase negative staphylococcus
Group B streptococcus
Staphylococcus aureus
Corynebacterium
Anaerobic cocci

References
Miller 5th Ed page 352

Question 25
Concerning Hip Biomechanics which of the following statements is FALSE?
A cane in the contralateral hand can reduce the joint reaction force by 60%.
Energy expenditure could be as high as 264% of normal with resection arthroplasty.
The ideal position of hip arthrodesis should be 25-30 degrees flexion and 0 degree of abduction and
rotation.
The joint reaction force can reach three to six times body weight due to muscular contraction.
Varus neck shaft angle of the hip increases joint reaction force and decreases shear across the neck..

References
Review

of

Orthopaedics,

Miller

5th

Edition.

A varus neck-shaft angle will decrease the joint reaction force but increase the shear force across the
neck

Question 26
A childs knee at the age of 3 years would normally show this alignment
Varus of 20 degrees
Varus of 10 degrees
Neutral alignment
Valgus of 10 degrees
Valgus of more than 20 degrees

References
Salenius P, Vankka E. The development of the tibio-femoral angle in children. JBJS Am 1975, vol 57,
pp
259

261.
During development children's tibio-femoral alignment changes during early years. At birth it is 10-15
degrees of varus, which remodels to neutral at about 24 months age and then becomes 10 degrees of
valgus at the age of 3 years. Over the next 4 years it gradually goes to normal alignment of 7 degrees.

Question 27
Which of the following operations is contraindicated when considering arthroplasty in a
rotator cuff deficient shoulder?
Bipolar hemiarthroplasty
Resurfacing hemiarthroplasty with subscapularis advancement
Resurfacing hemiarthroplasty without subscapularis advancement
Hemiarthroplasty with a smaller size than the native head
Anatomic total shoulder arthroplasty

References
Replacement of the glenoid is contraindicated in this situation. Abnormal mechanics in the cuffdeficient shoulder lead to eccentric loading of the superior glenoid, and rapid wear / loosening of the
component.

Question 28
Which one of the following is NOT the characteristic of nail-patella syndrome:
Hypoplasia of the nails with the thumb always affected
Cubitus valgus with limitation in extension and supination
Metatarsus adductus
Bony prominences from lateral surfaces of the ilium
Hypoplastic, wedge shaped and laterally displaced patella

References
Mercer's Orthopaedic Surgery, 9th edition, page no 147

Question 29
Which of the following does not form part of the plantar plate complex of the first MTPJ?
Joint capsule
Medial collateral ligaments
Lateral collateral ligaments

Tendons of flexor hallucis longus


Tendons of flexor hallucis brevis

References
Hallux
Valgus,
Hallux
varus,
and
sesamoid
disorders.
Jeffrey
A.
Mann
The plantar plate has a variety of roles in foot function. It is a fibrocartilaginous structure that aids in
stabilising the digit along with collateral ligaments, intrinsic and extrinsic muscles. It acts as an
attachment for the plantar fascia and has a role in the windlass mechanism. Its cartilaginous structure
helps to reduce compressive loads on the metatarsal heads. Combined with other adjacent structures
it also helps to guide the line of pull of various muscles attaching to the digits, specifically the
lumbricals and flexor digitorum longus

Question 30
Which of the following factors is least important when planning for revision of a failed total
knee replacement?
Bone loss and bone defects
Integrity of the extensor mechanism
Aetiology of the arthritis
Alteration of joint line
Attenuation of PCL

References
Miller.
Review
of
orthopaedics.
4th
ed.
Elsevier
2004
The major goals of revision knee surgery are 1) extraction of knee components with minimal bone and
soft tissue destruction 2) restoration of cavitory and segmental bone defects 3) restoration of the
original joint line as best as possible 4) adequate balancing of knee ligaments and 5) stable knee
components. The choice of knee implants also depends on the integrity of the MCL and PCL. If the
PCL is attenuated and/or the joint line remains significantly altered, a posterior stabilised implant is
recommended.

Question 31
In clavicle fractures treated non-operatively, risk of non-union is not increased by
Complete displacement
Male gender
Comminution
Advancing age
Lateral end fractures

References
Estimating the risk of non-union following nonoperative treatment of a clavicular fracture. Robindon
CM, Court-Brown CM, McQueen MM, Wakefield AE. JBJS Am 2004; 86: 1359-1365.
Although diaphyseal fractures account for the majority of non-unions encountered, this is because

clavicle fractures are most commonly seen in young, athletic individuals who commonly sustain
diaphyseal fractures skewing the data. Overall, non-union is most common in lateral end fractures
(11.5% vs 4.5% diaphyseal fractures and 8.3% medial end fractures) This paper demonstrated factors
that independently increased risk of non-union in non-operatively managed clavicle fractures which
were complete displacement (lack of cortical apposition) (relative risk=0.43), female gender (relative
risk= 0.70), presence of comminution (relative risk=0.69), and advancing age (relative risk=0.99).

Question 32
All of the following aspects are important in the surgical management of AVN of the femoral
head, using a vascularised fibula graft except
Core decompression
Curretage of necrotic bone from the drill hole
Revascularising the capsule
Placement of the fibular graft against the subchondral bone of the femoral head
Re-implanting a vascular supply to the graft.

References
Miller

Question 33
Subcuticular (continuous dermal) wound closure is
Contraindicated in patients prone to developing keloid
More resistant to exogenous bacterial contamination
Only suitable using absorbable sutures
Contraindicated if the wound is covered with a cast
A form of healing by secondary intention

References
Question 34
The detrusor muscle of the bladder is predominantly controlled by autonomic fibres with a
spinal root value of:
L1 and L2
L3 and L4
L5 and S1
S2 and S3
S4 and S5

References

Miller 4th Edition

Question 35
Platelet aggregation following endothelial injury is mediated by which of the following?
Protein C
Prothrombin
Von Willebrand Factor
Plasminogen
Factor VII

References
Question 36
Which of the following is true about collagen?
Procollagen is synthesised by myofibroblast
More than 90% of tendons and ligaments are made up of type II collagen
Type I collagen consists of three polypeptide chain
Collagen arrangement in ligaments is more parallel compared to tendons
Collagen content is greater in tendons than ligaments

References
Basic Orthopaedic sciences

Question 37
Which one of the following statements accurately describes the biopsy findings in patients
with Pagets Disease?
A significant decreased in the number and size of osteoclasts
Lace like mineralizing osteoid surrounding atypical osteoblasts
Poorly organized and chaotic structure with a mosaic pattern
Osteoblasts are present in large numbers within Howship's lacunae
Empty Lacunae with inflammatory cells predominate

References
Atlas
of
functional
Histology
Jeffrey
B
Kerr
Histological analysis of Pagetoid bone characteristically shows a mosaic pattern due to overacive
bone formation and resorption.. The trabeculae are thick and patterned by cement lines. Some
surfaces are excavated by osteoclastic activity whilst others are lined by rows of osteoblasts. The
marrow spaces contain fibrovascular tissue.

Question 38
When performing below knee amputation
A tourniquet should always be used
Skin flap length should be 50% of the AP diameter of the limb
The stump should be at least 6cm long to be functional
The fibula should be sectioned at least 5cm proximal to the tibia
The fibula should be excised routinely

References
http://www.orthoteers.com/(S(yeditm55daexvu45utzqa555))/searchresults.aspx?
section=35&article=260&searchterm=below+knee+amputation
Trauma.
Court
Brown
Lippincott
Williams
&
Wilkins
2006
p
259-264
Below knee amputations are the most common amputations carried out by orthopaedic surgeons.
There are 2 commonly used methods. The first is to use a long posterior myocutaneous flap and the
second involves 2 equal anterior and posterior flaps. The tibia should ideally be cut 12-15cm distal to
the knee joint and the fibula 1-1.5 cm shorter. However, modern prosthetic fitting allows for more
proximal resection of the tibia to the level just below the tibial tuberosity at around 6 cm distal to the
knee joint

Question 39
Factors associated with the formation of heterotopic ossification following acetabular fracture
include all of the following EXCEPT?
Head injury
Prolonged mechanical ventilation
Female gender
Extensile surgical approach
Extensive soft tissue injury

References
OKU-8: Chapter 32: Acetabular Fractures: Complications: Page: 395

Question 40
Which of the following cytokines is key in the pathogenesis of debris induced aseptic
loosening?
Transforming Growth Factor-Beta (TGF-)
Tumor Necrosis Factor-Alpha (TNF-)
Receptor Activator for Nuclear Factor B Ligand (RANKL)
Interleukin-1
Interleukin-6

References
The biology of aseptic osteolysis. Clin Orthop Relat Res. 2007 Jul;460:240-52

Question 41
Arthrogryposis is best described by which category of embryologic anomaly?
Failure of formation
Failure of differentiation
Duplication
Overgrowth
Undergrowth

References
Core Knowledge Orthopaedics - Hand, Elbow and Shoulder - page 424

Question 42
In the UK, the highest transmission risk from a packed red cell transfusion is:
HIV
Hepatitis C
Hepatitis B
CMV
Human T-cell lymphotropic virus (HTLV-1)

References
Miller,
5th
Edition
Hep C 1 in 1935000 Hep B 1 in 205000 HTLV-1 1 in 2993000 HIV 1 in 125000 CMV >70% donors
positive

Question 43
Select the correct statement regarding hypercalcaemia:
Approximately 10% of patients with primary hyperparathyroidism are asymptomatic.
Calcium levels in secondary hyperparathyroidism can be extremely high.
Hypercalcaemia in patients with squamous cell lung carcinoma is commonly due to a PTH-related
protein.
The most common cause of primary hyperparathyroidism is parathyroid carcinoma.
Chovstek's sign (tapping over the facial nerve causes twitching) is a reliable sign of hypercalcaemia.

References
Question 44
The Stainsby procedure to the lesser toes for subluxed/dislocated MTP joints involves
Open reduction of the MTPJ & metatarsal shortening
MTPJ fusion
Excision of the metatarsal head
Excision of the base of the proximal phalanx
Combined excision arthroplasty

References
Used Oxford Textbook of Orthopedics & Trauma, Orthoteers and Wheeless' online

Question 45
Which of the following is NOT part of the closed treatment of a Colles type distal radial
fracture?
Disimpaction
Exaggeration of the deformity
Reduction with ulnar deviation and wrist flexion
Locking the fracture by supination
Application of a radial slab with 3 point fixation

References
The

closed

treatment

of

common

fractures,

John

Charnley

Colles fracture has the following deformities- dorsal tilt, dorsal displacement, radial tilt and radial
displacement, shortening and supination. These deformities are reversed during closed manipulation.
The reduced position is held with the forearm in pronation. A dorso-radial slab is applied to maintain
reduction.

Question 46
The commonest cause of excessive growth hormone secretion is:
Eosinophilic adenoma within the anterior pituitary
Lung malignancy
Carcinoid tumour
Pancreatic islet-cell tumour
Renal osteodystrophy

References

Mercer's Orthopaedic Surgery, Nineth Edition, page 337

Question 47
Anterior transperitoneal approach to lumbar spine
Less extensive approach compared to extraperitoneal approach
Risk of retrograde ejaculation
Transverse incision should be made in front of the disc
Skin incision should always be longitudinal
Sympathetic plexus are not in the vicinity of the operating field

References
Campbells operative approach 8th edition

Question 48
Warfarin inhibits Vitamin K dependent factors by inhibiting the enzyme(s)
Vitamin K hydroxylase
Vitamin K epoxide
ATP Kinase
Vitamin K phosphorylase
All of the above

References
Question 49
Which of the following is considered to increase the likelihood of hip fracture following
femoral head resurfacing?
Valgus hip
Male sex
Low Body Mass Index
Surgical inexperience
Previous hip injection

References
Femoral neck fractures after metal-on-metal total hip resurfacing: a prospective cohort study. Marker
DR, Seyler TM, Jinnah RH, Delanois RE, Ulrich SD, Mont MA. J Arthroplasty. 2007 Oct;22(7 Suppl
3):66-71

Question 50

Which test is the most sensitive and specific for excluding deep infection prior to performing
single stage revision of the knee?
C-reactive Protein
Erythrocyte Sedimentation Rate
Isotope Bone Scan
Synovial Fluid Aspirate
White Blood Cell Count

References
Preoperative testing for sepsis before revision total knee arthroplasty. Della Valle et al. J Arthroplasty
2007:22,vol6,supp
1:90-93.
In the recent paper by Della Valle et al looking at a series of 105 consecutive painful total knee
replacements, a synovial fluid WBC count of greater than 3000 was found to be the most precise test
with a sensitivity of 100%, specificity of 98%, and accuracy of 99%. The preoperative use of an ESR
and CRP proved to be an excellent screening modality with only one infection identified with both
values being normal. A rational approach to perioperative testing for sepsis includes a screening ESR
and CRP, and if elevated, aspiration with synovial fluid WBC count or an intraoperative frozen section.

Question 51
The following statement regarding Slipped Upper Femoral Epiphysis is correct:
Occurs more frequently in girls
SUFE leads to delayed physeal fusion.
The slip occurs through the zone of hypertrophy
On AP plain radiographs, Hilgenreiner line passes superior to the head
The neck slips posterolateral in relation to the head.

References
1.
Apley's
System
of
Orthopaedics
(Arnold)
2. Miller, MD; Hart, JA: Review of orthopaedics, 5th Edition, Saunders 2008.
SUFE is more common in boys and on the left side. Once slipped, physis often closes early. It occurs
through the zone of hypertrophy, which is between the proliferation and maturation. Trethowan's sign
describes the fact the Klein's line passes above the head of femur in AP view. The neck of femur slips
anteriorly and superiorly in relation to the head.

Question 52
Following can be used to reconstruct chronic thumb MCPJ ulnar collateral ligament injury
EXCEPT:
Palmaris longus
Strip of fascia
Extensor pollicis brevis
Flexor pollicis brevis
Adductor pollicis advancement

References
Campbell's ninth edition Page 3390

Question 53
Enlargement of a vertebral body is the radiographic hallmark of one of the following:
Lymphoma
Metastatic spine disease
Eosinophilic granuloma of spine
Scheuermann's disease
Paget's disease

References
Principles

of

Orthopaedic

Practice.

Roger

Dee

et

al.

Second

edition.

P-60

This feature can be used to differentiate between Paget's and Lymphoma or metastasis of spine

Question 54
Your patient undergoes a hindquarter amputation for malignancy. The post operative course is
complicated by a wound infection. The causative organism(s)s is likely to be:
Staph aureus
Bowel commensals
Pseudomonas
Staph. epidermidis
Treponema pallidum

References
Hindquarter
amputation
SR
Carter.
DM
JBJS[Br]1990;72-B:490-3

for

tumours
Eastwood.

of
RJ

the

musculuoskeletal
Grimer.
RS

system.
Sneath

Question 55
In relation to the use of diathermy, which combination of current (Amps) and frequency(Hz) is
the most appropriate
50Hz, 100mA
50KHz, 500mA
1MHz, 500mA
1MHz, 100mA
10MHz, 5A

References
Question 56
Calcific tendinitis of the shoulder is:
More common in men
Bilateral in up to 50% of cases
Most common between 30 and 50 years of age
Divided into 5 typical stages
Associated with raised serum calcium levels

References
Calcifying
tendinitis
Hughes
PJ,
Bolton-Maggs
B,
Current
Orthopaedics
(2002)
16,
389-94
Calcifying tendinitis is more common in women, bilateral in upto 25% cases, is divided into 3 or
4stages(depends on what literature you read) but never 5 stages and serum calcium levels are not
elevated although ESR may be.

Question 57
A 40 year old patient was involved in a motorcycle accident and had a non salvageable foot.
He enjoys high-demand activities. Which prosthetic foot design would best suit him?
The solid ankle cushioned heel foot
The single-axis foot
Articulated dynamic response foot with a shortened keel
Articulated dynamic response foot with a long keel
Non-articulated dynamic response foot with a shortened keel

References
Miller, Review of Orthopaedics. Rehabilitation: Gait, Amputations, Prosthetics, Orthotics.

Question 58
The preferred surgical treatment of mild to moderate Boutonniere deformity (30 deg flexion
deformity of PIPJ with no arthritic changes in any of the joints) in a rheumatoid hand is:
Excision of the central slip of the extensor apparatus and PIPJ synovectomy
PIPJ fusion and transfer of extensor indicis proprius
DIPJ fusion and release of central slip
Release of the lateral slips with reconstruction of the central slip
Volar plate arthroplasty

References
Answer:

Ref:

Campbells

Operative

Orthopaedics

vol

4,

page

3699

A moderate buttonhole deformity has an approximately 40-degree flexion contracture of the proximal
interphalangeal joint, most of which is passively correctable. The distal joint is hyperextended, and
usually the metacarpophalangeal joint is correctable to full flexion passively. The lateral bands are
fixed in their subluxated position volarward by virtue of the contracted transverse retinacular ligament.
To correct this deformity there must be functional restoration of the central slip and correction of the
subluxation of the lateral bands. Roentgenograms of these joints should show no severe joint
destruction. If the metacarpophalangeal joint is destroyed and fixed but the interphalangeal joint is
preserved, this deformity can be treated with metacarpophalangeal joint arthroplasty or fusion.

Question 59
With regards to flexor tendon repair in the hand which of the following is true
Contraindication to repair includes active infection
Repairs are weakest between 15 and 30 days post-op
Sheath repair is mandatory
Zone 2 repairs have the best functional outcome
Zone 1 injuries are associated with concomitant neurovascular damage

References
Orthoteers
Miller. Review of Orthopaedics 4th edition

Question 60
During orthopaedic surgical dissection what substances are released that is involved in the
pathogenesis of deep vein thrombosis
Thromboplastin
Antithrombin III
Prothrombin
Factot X
Factor XII

References
Question 61
Five days following a fall down a flight of stairs a 65 year old man has reduced sensation and
MRC grade 3 power in his lower limbs with grade 0 power and absent sensation in his upper
limbs. Muscle tone is raised in his lower limbs but reduced in his upper limbs. Peri-anal
sensation is present. His spinal cord lesion is most likely to be:
Complete cord lesion

Anterior cord syndrome


Central cord syndrome
Posterior cord syndrome
Brown-Sequard syndrome

References
Millar
4th
Apley's system of Orthopaedics and fractures 8th Edition.

Question 62
Which element is responsible for the corrosive resistance of stainless steel?
Nickel
Carbon
Chromium
Manganese
Iron

References
Ramachandran Basic Sciences page 156

Question 63
With regards to elbow instability which of the following statements is true?
Soft tissue disruption progresses from medial to lateral in elbow dislocation.
Biceps is the most important dynamic stabiliser of the elbow joint.
Lateral pivot shift test is positive in posterolateral rotator instability.
Does not follow tennis elbow release.
Coronoid is not an important part of the force bearing surface of the elbow.

References
Question 64
In an intrinsic plus hand one of the following is true:
With the MCPJ completely extended, the PIPJ cannot be flexed
With the MCPJ completely flexed, the PIPJ cannot be extended
With the PIPJ completely extended, the MCPJ cannot be extended
With the PIPJ completely flexed the MCPJ cannot be extended
With MCPJ completely flexed, PIPJ cannot be flexed

Edition

References
Answer:
a:
Ref
Campbells
Operative
Orthopaedics
vol
4,
page
3694
The intrinsic plus deformity is caused by tightness and contracture of the intrinsic muscles. In hands
with intrinsic plus deformity the proximal interphalangeal joint cannot be flexed while the
metacarpophalangeal joint is fully extended. Often the deformity develops in combination with volar
subluxation of the metacarpophalangeal joints and ulnar deviation of the fingers. To perform the
Bunnell test for intrinsic tightness, the metacarpophalangeal joint is passively held in extension,
causing passive extension of the proximal interphalangeal joint and preventing passive flexion of the
proximal interphalangeal joint. When the metacarpophalangeal joint is passively flexed, the intrinsics
are relaxed and passive flexion of the proximal interphalangeal joint is increased.

Question 65
Anterior approach to the hip utilises which of the following deep muscular planes to gain
access to the joint capsule.
Sartorius and adductor longus
Sartorius and rectus femoris
Tensor fascia lata and rectus femoris
Rectus femoris and gluteus medius
Rectus femoris and gluteus minimus

References
The deep internervous muscular plane lies between rectus femoris (femoral nerve) and gluteus
medius (superior gluteal nerve). The superficial plane lies between Sartorius (femoral nerve) and
tensor facia lata (superior gluteal nerve).

Question 66
A volar V-Y advancement flap is recommended for soft tissue reconstruction of a finger tip
injury
Dorsal oblique amputation where the defect does not exceed 1 cm
Dorsal oblique amputation where the defect does exceed 1 cm
Transverse amputation where the defect does exceed 1 cm
Volar oblique amputation where the defect does not exceed 1 cm
Volar oblique amputation where the defect does exceed 1 cm

References
Reconstruction of the amputated finger tip with a triangular volar flap: A New Surgical Procedure
Atasoy
et
al
JBJS(Am)1970;52:921-926
2.
Selected
readings
in
plastic
surgery.
vol.9
No.32
2002
(p20-21)
Indications are for transverse or dorsal oblique patterns of amputation where the defect does not
exceed 1 cm.

Question 67
The Anterior Interosseous Nerve:
Originates as a branch of the radial nerve
Runs on the volar surface of Flexor digitorum profundus
Innervates pronator teres
Provides cutaneous supply to the anterolateral surface of the forearm
Innervates flexor pollicis brevis

References
Last's
anatomy
Arises from median nerve, 5 cm above medial epicondyle; runs on volar surface of FDP and along
interosseous membrane between ulna & radius; supplies FPL, lateral half of FDP, & pronator
quadratus; may supply sensory branches to distal radio-ulnar and carpal joints.
Martin Gruber anastomosis occurs in 10-15% of all forearms and in half of these cases, the nerve
communication arises from the AIN branch; hence palsy of the AIN could lead to palsy of the hand
intrinsics normally supplied by the ulnar nerve

Question 68
Which of the following is NOT a pre-requisite for tendon transfer?
None of the below
Muscle power of tendon atleast Gr.IV
Antagonists preferred over Agonists
Smooth course of action
Phasic rather than Non-phasic transfer

References
Question 69
The central portion of the plantar aponeurosis originates from the following :
Medial tuberosity of the os calcis
The lateral tuberosity of the os calcis.
The sustentaculum tali
The abductor digiti quinti
Abductor hallucis muscles

References
Surgery of the Foot and Ankle.Coughlin and Mann.Mosby.1999.

Question 70

The most likely indication for a spica cast as the definitive treatment of a paediatric femoral
fracture would be:
A child between 18 months to 5 years of age with a simple fracture pattern
A child between 18 months to 5 years of age with a complex fracture pattern
A child up to 5 years of age with multiple injuries
An infant up to 18 months of age with a simple fracture pattern
An infant up to 18 months of age with a complex fracture pattern

References
Anglen JO, Choi L. Treatment options in pediatric femoral shaft fractures. J Orthop Trauma.
2005;19:724733

Question 71
A Mason II radial head fracture is present if
There is less than 30% articular involvement and more than 2 mm of displacement
There is an anterior fat pad anterior to the coronoid fossa and displacement of the fracture of less
than 2 mm
There is an articular fracture involving the entire head, that consists of more than two large fragments
that are separated from the shaft
There is a fracture of the entire radial head with displacement from the shaft
There is a tilted and impacted articular segment and some of the articular fragments are displaced
from the shaft

References
Mason
ML.
Some
observations
on
fractures
of
with a review of one hundred cases. Br J Surg 1954;42:123-32.

the

head

of

the

radius

Question 72
Epineural Neurorrhaphy:
Primarily involves suturing of the fasciculi
Is commonly used for repair of nerve injuries sustained in association with closed fractures
Can only be safely performed in the first 24 hours after injury
Can not be used alone when long segments of nerve have been traumatised or excised
Requires a segment of nerve (such as the sural nerve) to be harvested

References

Cambell's
operative
orthopaedics
Epineural repair cannot be used when long segments have been lost because other techniques such
as grafting or transposition would be required to achieve a tension free repair.

Question 73
With regards to fractures of the distal radius, which of the following is true
The Cotton-Loder position (excess flexion & ulnar deviation) is associated with high carpal tunnel
pressures.
A below elbow cast in extension & ulnar deviation is commonly used.
Late tendon rupture is related to initial fracture displacement.
90% of patients following MUA and cast treatment of a distal radius fracture have full movement and
grip strength at 1 year.
In the application of a below elbow cast the ulnar nerve is commonly affected secondarily to cast
pressure.

References
kozin, wood, JBJS 1993;75A:144-152

Question 74
Regarding lower limb fasciotomy for compartment syndrome. Which of the following is NOT in
the anterior compartment?
Deep peroneal nerve
Extensor digitorum longus
Extensor hallucis longus
Peroneus brevis
Peroneus tertius

References
Surgical

Exposures

in

Orthopaedics,

3rd

Ed.

Hoppenfeld

&

deBoer

The anterior compartment consists of the following- Tibialis anterior, extensor digitorum longus,
extensor hallucis longus, peroneus tertius, deep peroneal nerve and anterior tibial artery. Peroneus
brevis is a content of the lateral compartment.

Question 75
What type of collagen is unique to intervertebral discs?
Type V
Type VI
Type VII
Type VIII

Type IX

References
Basic
Orthopaedic
First
Edited
by
Published
by
Chapter 15 Page 136

Sciences,

The

Stanmore

Manoj
Hodder

Arnold,

Guide
Edition
Ramachandran
London

Question 76
Which of the following clinical and radiological features is NOT consistent with static reducible
scapholunate dissociation?
A palpable clunk on Kirk Watson's test
A scapholunate angle of 50 degrees on a lateral radiograph
A scapholunate interval of 4mm on the AP radiograph
An apparent foreshortening of the scaphoid on the AP radiograph
An extended posture of the lunate on radiographs

References
Greens
Operative
Hand
Surgery.5th
Edition
pp.555-556.
Scapholunate dissociation is the most common carpal instability. Scapholunate instability is
associated with increased scaphoid flexion and pronation with associated lunate extension. The
abnormal kinematics leads to a decrease in surface area contact at the radioscaphoid joint. This
abnormal articulation causes an increased concentration of load, leading to the development of
degenerative arthritis. The patient may give a history of clicking and clunking of the wrist. On
examination, there is tenderness about the scapholunate interval, which lies just distal to the Lister
tubercle. Provocative manoeuvres for scapholunate instability, such as the scaphoid shift test, may be
positive and there is often associated grip strength weakness. Radiographs show in AP view "Terry
Thomas"sign gap between scaphoid and lunate normal 1-2 mm (abnormal 3mm) Progressive flexion
and foreshortening of the scaphoid leads to the scaphoid ring sign, seeing scaphoid end on. The
normal lateral scapholunate angle is 45-60 degrees.

Question 77
Which of the following structures is the most common origin for a volar ganglion in the hand
and wrist?
The scapholunate ligament
The triangular fibrocartilage complex
The flexor retinaculum
The distal radioulnar joint capsule
The scaphotrapezial joint capsule

References

Review
of
Orthopaedics,
Miller
Chapter
7
Answer E The scaphotrapezial joint capsule The most common hand/wrist ganglion is the dorsal wrist
ganglion typically arising from the scapholunate ligament. The second most common hand/wrist
ganglion is the volar wrist ganglion, typically arising from either the radioscaphoid joint or
scaphotrapezial joint capsule.

Question 78
De Quervain's disease
Typically occurs in young adults
Is more common in men
Almost always related to overuse
Thickening of fibrous sheath is never palpable
Involves tendon of Adductor pollicis longus.

References
Operative
Orthopaedics,
Campbell,
10th
ed
Stenosing tenosynovitis of the abductor pollicis longus and extensor pollicis brevis tendons occurs
typically in adults between 30 and 50 years old. Women are affected 10 times more frequently than
men. The cause is almost always related to overuse, either in the home or at work, or is associated
with rheumatoid arthritis. The presenting symptoms usually are pain and tenderness at the radial
styloid. Sometimes a thickening of the fibrous sheath is palpable. The Finkelstein test usually is
positive: "on grasping the patient's thumb and quickly abducting the hand ulnarward, the pain over the
styloid tip is excruciating."

Question 79
The following technique for repair of distal biceps tendon rupture is the strongest (highest
load to failure)
Biceps brachii tenodesis
Suture anchor repair
Interference screw
Bone tunnel technique
Endobutton technique

References
Reattachment
of
the
distal
tendon
of
biceps
FACTORS
AFFECTING
THE
FAILURE
STRENGTH
OF
THE
REPAIR
Journal of Bone and Joint Surgery - British Volume, Vol 90-B, Issue 1, 103-106.
doi: 10.1302/0301-620X.90B1.19285 Augustus et al : Biomechanical evaluation of four techniques of
distal biceps brachii tendon repair;Am Journal of Sports MedicineFeb 2007 35(2): 252-258
Operative fixation is the treatment of choice for a rupture of the distal tendon of biceps. A variety of
techniques have been described including transosseous sutures and suture anchors.
The Endobutton-based method showed the highest failure load at 270 N (SD 22) (p < 0.05). The
mean failure load of the transosseous suture technique was 210 N (SD 66) and that of the TwinFixQuickT 5.0 mm was 57 N (SD 22), significantly lower than those of all other repairs (p < 0.05). No

significant correlation was seen between bone mineral density and loads to failure.
The transosseous technique is an easy and cost-saving procedure for fixation of the distal biceps
tendon. TwinFix-QuickT 5.0 mm had significantly lower failure loads, which might affect early
rehabilitation, particularly in older patients.

Question 80
The most important step in the initial management of an open long bone fracture
Airway, breathing and circulation
IV antibiotics
Covering the wound with Betadine soaked swab
Photograph of the wound
X ray of the fracture

References
Question 81
Which of the following statements about the sequelae of Slipped Upper Femoral epiphysis is
true:
Patients who develop chondrolysis present with no pain or decreased motion
If chondrolysis occurs, degenerative changes are less likely to develop.
Degenerative joint disease is characterised by a pistol grip deformity of the proximal femur.
Limb length inequality is unlikely to develop in cases of incomplete reduction.
Acetabular dyslasia is a common sequelae of SUFE.

References
Miller,
MD;
Hart,
JA:
Review
of
orthopaedics,
5th
Edition,
Saunders
2008.
SUFE, even when treated, can be associated with significant complications such as chondrolysis,
pistol grip OA and unequal limbs. Chondrolysis is associated with severe pain and movement
restriction.

Question 82
Following core decompression for early non-traumatic osteonecrosis of the femoral head
(Ficat Stage I or II) a good result in terms of Harris Hip score or radiographic failure of
progression can be expect in what percentage of patients?
35-45%
45-55%
55-65%
65-75%
75-85%

References
Jay R. Lieberman, Daniel J. Berry, Michael A. Montv, Roy K. Aaron, John J. Callaghan, Amar
Rayadhyaksha,
and
James
R.
Urbaniak
Osteonecrosis
of
the
Hip:
Management
in
the
Twenty-first
Century
J.
Bone
Joint
Surg.
Am.,
May
2002;
84:
834
853.
Based on two extensive literature reviews by Smith and Mont et al respectively approximately 70% of
patients improved

Question 83
Aging is associated with changes in mechanical properties of articular cartilage. With
increasing age the following changes in articular cartilage mechanical properties are noted
EXCEPT
Increase in stiffness
Decrease in strength
Decrease in fatigue resistance
Decrease in strength
Decrease in brittleness

References
Ulrich-Vnther
M,
Maloney
M,
Articular cartilage Biology. J Am Acad Orthop Surg 2003; 11:421-430

Scwartz

EM

Question 84
In a child diagnosed with Graf Grade 3 developmental dysplasia of the hip at ultrasound, which
one of the following anatomical abnormalities are present?
High dislocation with a shallow acetabulum
High dislocation with a normal acetabulum
Immature hip only in children under 3 months of age
Mild dysplasia in children over 3 months of age
Normal hip pathology

References
Graf R. The ultrasound examination of the hip. In: Tonnis D, ed. Congenital dysplasia and dislocation
of
the
hip.
Berlin:
pringer,
1987:172-229.
Dislocation with a shallow acetabulum is grade three

Question 85
What is the most frequent morbidity associated with local pedicle flaps in the digits?
Pain

Infection
Paraesthesia
Joint stiffness
Swelling

References
HAND
SECRETS
The most common morbidity is joint stiffness. This is because pedicle flaps require 10-14 days of
immobilisation, resulting in stiffness.

Question 86
What is the primary goal in the treatment of Legg-Calve-Perthes disease?
Maintaining good range of movement
Containing the femoral head within the acetabulum
Maintaining non weightbearing throughout the active course of the condition
Early surgical core decompression of the femoral neck
Maintaining the neck shaft angle of the femur

References
Review
of
orthopaedics,
Campbells Operative Orthopaedics, ed. Canale, 9th ed

Miller

4th

ed

Question 87
What is the primary stabiliser of the elbow to varus stress in the extended position?
Lateral ulnar collateral ligament
Medial ulnar collateral ligament
Anterior capsule
Radio-humeral joint
Ulno-humeral joint

References
Basic
Orthopaedic
sciences
,The
Stanmore
guide,
M
Ramachandran,
pg
195
In varus stress the primary stabiliser is the Ulno-humeral joint while the anterior capsule acts as
secondary stabiliser with elbow in extension and the lateral collateral ligament acts as secondary
stabiliser with elbow in flexion.

Question 88

A 42 year old man has a 1 year history of lower thoracic back pain. For the last 2 months, he
has been having shooting pain down his legs. Examination reveals upper motor neuron signs
along with signs of weakness in the legs. A sagittal MRI scan shows a T11-12 posterolateral
disc prolapse. The most appropriate treatment for this would bePosterior decompression alone
Posterior decompression and fusion
Trans-thoracic anterior decompression
Thoracolumbar orthosis
Bed rest and gradual mobilisation

References
Bohlman & Zdeblik:Anterior excision of herniated thoracic discs. J Bone and Joint Surg Am,70:10381047,1988.
The correct answer is 3- transthoracic anterior decompression. Conventional posterior decompression
+/- fusion is inappropriate as any posterior approach would require mobilsiation of the spinal cord to
one side, which might not only damage the blood supply but also would provide an inadequate
exposure of the disc. The presence of myelopathic signs along with weakness in a 42 year old would
justify a surgical approach.

Question 89
Quadriceps tendon repair
May be performed using the MacIntosh technique
Is most commonly necessary in the 4th decade of life
Often results in loss of range of motion and muscle strength when compared to the non-injured leg
Does not occur in association with a patellar spur
Does not occur in association with an open bicondylar Hoffa fracture of the knee

References
1. Schlatterer B, Jund S, Delpine F, Razafindratsiva C, de Peretti F.[Acute anterior cruciate ligament
repair with combined intra- and extra-articular reconstruction using an iliotibial band with the modified
MacIntosh technique: a five-year follow-up study of 50 pivoting sport athletes]Rev Chir Orthop
Reparatrice
Appar
Mot.
2006
Dec;92(8):778-87.
2. Puranik GS, Faraj A.Outcome of quadriceps tendon repair.Acta Orthop Belg. 2006 Apr;72(2):176-8.
3. Ramseier LE, Werner CM, Heinzelmann M.Quadriceps and patellar tendon rupture.Injury. 2006
Jun;37(6):516-9.
Epub
2006
Feb
23.
4. Hardy JR, Chimutengwende-Gordon M, Bakar I.Rupture of the quadriceps tendon: an association
with
a
patellar
spur.J
Bone
Joint
Surg
Br.
2005
Oct;87(10):1361-3.
5. Calmet J, Mellado JM, Garca Forcada IL, Gin J.Open bicondylar Hoffa fracture associated with
extensor mechanism injury.J Orthop Trauma. 2004 May-Jun;18(5):323-5.

Question 90
Which of the following ligaments have a lowest load to failure (N)?
Anterior longitudinal ligament

Posterior longitudinal ligament


Ligamentum flavum
Interspinous ligament
Capsular ligament (facet joint)

References
White AA, Panjabi MM. (1990). Clinical biomechanics of the spine. Philadelphia, JB Lippincott.

Question 91
Criteria for adequate resuscitation of a polytrauma patient include all of the following EXCEPT
Hemodynamically stable without need for inotropic support
Lactate level > 2 mmol/L
No coagulopathy (INR < 1.25)
Urine output > 1 mL/kg/h
Normal temperature

References
Ref: SURGICAL PRIORITIES IN DAMAGE CONTROL IN POLYTRAUMA. Giannoudis, P. V. Journal
of Bone & Joint Surgery. 85-B(4):478-483, May 2003.

Question 92
Which of the following is true for the knee joint
The ratio of rolling to gliding remains constant through all degrees of flexion
The lateral meniscus is larger in diameter and less mobile than the medial
In the fully extended knee rotatory movements are still possible
The magnitude of femoral external rotation is greater in an unloaded knee
On loading the knee, lateral meniscus carries all the load and the medial meniscus shares it equally
with the articular cartilage.

References
Question 93
With regards to stand alone anterior lumbar interbody spinal fusion following is true
Nonunion rate is less than 1%
Contraindicated in smokers
Osteoporosis is a relative contraindication
They work better for two level fusion than single level fusion

Provides adequate stability for high grade lumbar spondylolisthesis

References
Campbells
operative
Question Comments

orthopaedics

8th

edition

and

contemporary

practice

Question 94
In the treatment of Developmental Dysplasia of the Hip, in what position should the legs be
placed in a Pavlik Harness?
45 degrees flexion and 45 degrees abduction
45 degrees flexion and mild abduction
100 degrees flexion and mild abduction
100 degrees flexion and 45 degrees abduction
mild flexion and mild abduction

References
Review Of Orthopaedics, Miller, 54th ed. Page 226

Question 95
The size of wear particles that are biologically active in producing wear is
10 100 micrometer
0.1 10 millimeter
10 100 nanometer
0.1 - 10 micrometer
0.1 0.5 millimeter

References
Friction, Lubrication and Wear of Artificial Joints, Ian M. Hutchings (edt), 2003, John Wiley and Sons,
p 23

Question 96
Regarding flexor tendon injuries in the hand:
FDS excision may result in Swan Neck deformity
Zone 1 injuries involve only the FDS tendon
In zone 2 injuries, both FDS and FDP tendons should always be repaired
Active flexor tendon loading should commence within 2 weeks

Repair should be performed in all partial flexor tendon injuries involving 20% or more of the tendon

References
Wheeless
Online
Textbook
of
Orthopaedics:
http://www.wheelessonline.com/ortho/flexor_tendon_repair accessed 2 December 2009

Question 97
Which is the lowest grade of shock one would expect to find a drop in systolic blood pressure
in a 25 year old healthy male following major trauma?
Grade 1
Grade 2
Grade 3
Grade 4
Grade 5

References
ATLS
Manual
8th
edition
Hypovolaemic shock is graded according to severity. This is measured by different clinical signs
namely heart rate, respiratory rate, blood pressure, urine output and cognitive function. Class 1
(<750mls/ <15% blood volume lost) HR <100, BP normal, RR 14-20, UO >30, CNS slightly anxious.
Class 2 (750-1500mls/ 15-30% blood volume lost) HR >100, BP normal, RR 20-30, UO 20-30, CNS
mildly anxious. Class 3 (1500-2000mls/ 30-40% blood volume) HR 120, BP decreased, RR 30-40, UO
5-15, CNS anxious and confused. Class 4 (>2000mls/ >40% blood volume) HR >140, BP decreased,
RR>35, CNS confused, lethargic.

Question 98
The treatment of choice for a periprosthetic femoral fracture classified as Vancouver B1 is
Revision with longer stem prosthesis
Cerclage wire fixation
Non-surgical management
Open reduction and extramedullary fixation
Closed reduction and intramedullary nailing

References
Campbell P, McWilliams TG. Periprosthetic femoral fractures. Current Orthopaedics 2002;16:126-132.
The Management of Periprosthetic Femoral Fractures around Hip Replacement Fares S. Haddad,
BSc, MCh (Orth) FRCS (Orth) A L: Lesser Trochanter. A rare fracture following total joint arthroplasty.
It is usually avulsion in type, occurring in osteopenic bone or through an osteolytic lesion.
Symptomatic treatment alone will usually suffice, although the presence of substantial calcar fragment
may demand cerclage wiring or revision hip arthroplasty if the stability of the implant is lost. AG:
Greater Trochanter This is typically an avulsion injury in osteopenic bone. If only minimally displaced
then the composite tendons of the glutei and vasti remain intact and prevent further displacement.

Symptomatic treatment with the use of crutches and avoidance of active abduction is usually sufficient
to allow healing. However, if there is substantial displacement then open reduction and internal
fixation need consideration to prevent non-union and its associated complications. Without previous
disruption of the aponeurosis of the abductors and vastus lateralis, it is uncommon for these injuries to
severely displace. If there is polyethylene wear and severe osteolysis, the primary problem may need
to be addressed. Type C:- Shaft fracture well below the femoral component These fractures can be
managed as conventional femoral fractures. Surgical intervention is frequently the rule as it allows
rapid mobilization and prevents prolonged hospitalisation. We have recently found LISS plates
particularly useful for these fractures. Type B: Shaft fracture around or close to the stem tip - These
are by far the most common periprosthetic fracture (80% in both European and North American
series). B1. Implant stable - when the femoral component is stable, open reduction and internal
fixation is recommended in all but exceptional cases. Non-operative treatment is associated with the
well-recognized complications of prolonged bedrest, and may result in malunion. Malunion, in turn,
may lead to functional problems, and makes subsequent revisions more difficult. Revision of the stem
poses problems as removal of a well-fixed implant is technically challenging, and can result in
considerable bone loss. Attention has therefore centered on the use of internal fixation to treat
periprosthetic femoral fractures around stable implants. Screws, cerclage wires, cables or bands on
their own are insufficient, and conventional plates commonly fail because of the difficulty in obtaining
proximal fixation. Moreover, proximal screws may violate the bone prosthesis interface, may lead to
cement fracture and loosening, and will act as stress risers increasing the risk of later fractures.
Alternatives have included combinations of plates and cables, Partridge bands with a variety of plates,
Mennen plates and compression plates, some of which are associated with high failure rates. Cortical
onlay strut allografting (with or without adjunctive plates) has emerged as a very attractive option for
periprosthetic femoral fractures around stable implants. The strut allografts act as biological bone
plates that serve both a mechanical and a biological function. The allograft struts confer stability to the
fracture site, and can incorporate and ultimately increase the femoral bone stock. B2. Implant loose when the femoral implant is already loose then both the fracture and the loose prosthesis should be
addressed. The prosthesis should be revised using a mid or long stem implant. This may be
augmented with a cortical strut graft depending on the surgeons preference. Cemented or
cementless fixation of the component will depend on the philosophy of the surgeon and the centre as
well as the physiologic age of the patient and their femoral anatomy. In the elderly patient where
immediate full weight bearing is desirable a cemented prosthesis is recommended. Impaction grafting
has also been shown to be a useful technique in this setting. B3. Implant loose and inadequate bone
stock - when the implant is loose and there is inadequate bone stock around the prosthesis then a
more complex reconstruction may be required. Distal fixation with or without proximal femoral
reconstruction is usually necessary. Augmentation of the bone may require intramedullary morsellised
bone along with cortical onlay allograft struts, substitution of the proximal deficient bone stock by a
segmental replacement allograft, or substitution of the proximal femur with a modular oncology
prosthesis have been the traditional options. More recent alternatives include the use of tapered
stems and of locked implants

Question 99
Which one of the following hormones delays fracture union by decreasing callus proliferation?
Growth hormone
Cortisone
Thyroid Hormone
Parathyroid Hormone
Calcitonin

References
Miller's
review
of
Orthopaedics.
4th
Edition
.Page:
19
Cortisone leads to decreased callus proliferation. Calcitonin has a positive effect on fracture healing.

The mechanism is unknown. Thyroid hormone and parathyroid hormone increase bone remodelling
and growth hormone increases callus volume.

Question 100
Which of the following is not true about metabolic intervention in trauma
Branched chain amino-acids improve the retention of nitrogen after trauma
Early enteral feeding after blunt trauma attenuates the stress response
Exogenous glutamine administration helps to maintain intestinal integrity and enhances immune
function
Recombinant human growth hormone improves nitrogen balance after trauma
Beta-2 adrenoceptor agonists have not been proven to be beneficial in attenuating the catabolic
response in the flow phase

References
1)

Eyer

2)Cerra

SD
et

et
al.

al.
Annals

Journal

of

of

trauma
surgery

1993;
1984;

34(5):639-43
199(3):286-91

3) Griffiths et al. British medical bulletin 1999; 55(1)

Question 101
A 32 year old male complains of pain and swelling on the ulnar aspect of left thumb MCP joint
following a skiing accident. Plain x-rays show an avulsion fracture of the base of proximal
phalanx on the ulnar side with <2mm displacement. What is the most appropriate way to
manage this injury?.
Perform stress x-rays to check for instability and then plan treatment
Treat in a thumb spica for 4-6 weeks
Excise the fragment and do a direct repair of UCL to bone
Directly reattach the bony fragment
Perform a reconstruction of UCL using forearm fascia

References
1) Canale: Campbell's Operative Orthopaedics, 10th edition. 2) Browner: Skeletal Trauma: Basic
Science, Management, and Reconstruction, 3rd ed. 3) Fricker R, Hintermann B. Skier's thumb.
Treatment, prevention and recommendations. Sports Med. 1995 Jan;19(1):73-9. A minimally
displaced (=2 mm) avulsion fracture signifies a complete avulsion without a Stener lesion. This usually
heals with casting for 4-6 weeks. To prevent the development of a Stener lesion, the joint should not
be stressed. If a Salter-Harris type I or type II fracture is present in a child, stress films are
contraindicated.

Question 102

The main advantage of treating a dorsally angulated distal radius fracture using a volar locking
plate compared to closed reduction and per-cutaneous intra-focal pinning in elderly patients
is?
Improved long term functional result
Less number of complication
Better intra-operative anatomic reduction
Better maintenance of achieved reduction
Better longterm patient satisfaction

References
J
Hand
Surg
[Am].
2007
Nov;32(9):1385-92.
Links
A comparative study of clinical and radiological outcomes of dorsally angulated, unstable distal radius
fractures in elderly patients: intrafocal pinning versus volar locking plating.Oshige T, Sakai A, Zenke Y,
Moritani
S,
Nakamura
T
Locking plate has been shown to maintain the reduction more effectively than k-wires in osteopenic
bones in the elderly. The above mentioned study compared the clinical and radiological outcomes of
intrafocal pinning (IFP) and volar locking plating (VLP) of dorsally angulated, unstable distal radius
fractures in elderly patients. They studied 62 consecutive patients over 60 years of age with dorsally
angulated, unstable distal radius fractures treated with IFP or VLP. Bone mineral density (BMD) of the
lumbar spine was measured. The range of motion and grip strength were measured at follow-up
examinations, and ulnar variance (UV) was measured on radiographs at baseline and follow-up
postoperative examinations. They found that VLP, but not IFP, can maintain surgically corrected UV in
distal radius fractures, independent of the degrees of initial UV and BMD. VLP also enhanced earlier
recovery in range of motion and grip strength than IFP.

Question 103
Which of the following statements is true with regards to paediatric elbow fractures?
Ulnar nerve is the most commonly affected nerve in extension type supracondylar fractures.
Pucker sign suggests an undisplaced fracture.
Absent anterior fat pad sign is most reliable in ruling out a fracture.
Normal humeral shaft condylar angle is 80 degrees.
Tardy ulnar nerve palsy is a known complication of old undetected radial head dislocation.

References
Question 104
The integrity of which of the following does Elson's test demonstrate?
The FDP tendon
The central slip of the extensor apparatus
The lateral extensor bands
The lumbrical muscles

The sagittal bands

References
The test described by Elson demonstrates integrity of the central slip - Put finger over edge of table,
with PIPJ flexed to 90deg. and ask the patient to extend against resistance. Weakness of resisted
extension of PIPJ & hyperextension of DIPJ occurs if the central slip is ruptured.

Question 105
Regarding rotator cuff tendinopathy of the shoulder
Bursal surface tears have poor blood supply compared with articular tears
Tendons heal by collagen synthesis followed by fibroblast formation
Asymptomatic cuff tears are uncommon
Gerber's "lift-off" test assesses the functional integrity of infraspinatus
Recurrent anterior dislocation of the shoulder may cause disruption of subscapularis

References
Miller 4th ed, review of orthopaedics Campbells operative Orthopaedics 11th edition
Tendon healing is initiated by fibroblasts(miller 4th ed pg 82) Rotator cuff pathology is a common
problem, and cadaver anatomical studies have reported rotator cuff tears in 30% to 50% of
specimens, suggesting that they may be part of the normal aging process(campbells 11th ed, pg
2615-2619) Gerbers lift off test assesses subscapularis and not infraspinatus.There is no difference
in the blood supply on the bursal or articular surface. With regard to recurrent anterior dislocation,
Asynchronous fatigue of the rotator cuff from overuse or incompetent ligamentous support can result
in further damage to the static and dynamic supports. MRI studies have shown fatty infiltration and
thinning of the subscapularis tendon in recurrent anterior instability(Campbells operative orthopaedics
11th ed, pg 2677).

Question 106
In sensory recovery after nerve repair, the most important factor is;
Associated injuries of muscles, tendons and bone
Age of the patient
Primary repair of injured nerve
Tension free repair of the injured nerve
Level of injury, better outcome with proximal injury

References
Campbells operative orthopaedics. Ed; S Terry Canale. Tenth edition, 2003, pp 3530.
In sensory recovery after nerve repair, the most important factor is age of the patients. Patients under
the age of 20 year old have better can be expected to have a better prognosis than older patients.
Tension free repair of the injured nerve is required for better outcome. , Better outcome is expected

with more distal injury. More severe associated injuries of the surrounding tissue has an adverse
effect on nerve recovery.

Question 107
A 40 year old male cross-country runner develops knee & back pain after increasing his
weekly training mileage. Your assessment finds him to have moderate overpronation of both
feet as evidenced by calcaneo-valgus. You recommend Softer / more cushioned training shoes.
A soft hindfoot orthosis.
A hard hindfoot orthosis
A viscoelastic hindfoot orthosis
Harder / less cushioned training shoes.

References
De Lee & Druz

Question 108
Which of the following is NOT typically associated with osteonecrosis of the femoral head?
Radiation therapy
Heavy alcohol intake
Rheumatoid arthritis
Sickle cell disease
Steroids

References
Review of Orthopaedics. 4th edition. Edited by Mark D. Miller. p41-42

Question 109
In a clinical study involving stabilisation of periprosthetic fracture of Total knee arthroplasty
with LISS plate and retrograde nailing, the LISS plate is better than nail with regards to...
Less operating time
Reduced length of stay
Better fixation in osteoporotic fracture
Minimally invasive procedure
Joint damage

References

Wick M, Muller EJ, Kutscha-Lissberg F et al.[Periprosthetic supracondylar femoral fractures: LISS or


retrograde intramedullary nailing? Problems with the use of minimally invasive technique].
Unfallchirurg 2004;107(3)181-188

Question 110
With regards to fractures of the acetabulum and pelvis:
A : Anteroposterior compression injuries with symphysis widening >2cm and ant/post SI ligament
rupture.
B : Both anterior and posterior columns.
C : The acetabular dome
D : The anterior column.
E : The anterior column and posterior acetabular wall.
F : The posterior column
G : The posterior column and anterior acetabular wall
H : Unilateral rami fracture with ipsilateral fracture of posterior iliac crest
1 : The structures best visualised by the oblique obturator Judet view.
Correct answer:
E
Your answer:
E
2 : Has the highest association with major haemorrhage.
Correct answer:
A
Your answer:
A
3 : Fractures associated with the poorest functional outcome
Correct answer:
B
Your answer:
A

References
Question 111
For the following scenarios please choose the most appropriate arthroscopic procedure.
A : Lateral release
B : Meniscal repair
C : Microfracture
D : Partial meniscectomy
E : Removal loose body
F : Synovectomy
G : Total meniscectomy
H : Arthroscopic washout
I : MACI
J : Hoffas fat pad excision
1 : A 55 year old patient with early rheumatoid arthritis and severe knee pain uncontrolled with
conservative treatment
Correct answer:
F
Your answer:
F
2 : A 23 year old sportsperson with a longitudinal red zone meniscal tear.
Correct answer:
B

Your answer:
B
3 : A 30 year old female with 1cm chondral defect on the medial femoral condyle.
Correct answer:
C
Your answer:
C

References
Question
References
Apleys System of Orthopaedics and Fractures. Solomon, Hodder Arnold 2001 Lateral Release for
Recurrent Dislocation of the Patella.Dandy D.J, Griffiths,G. J Bone Joint Surg Br 1989;71-B:121-5
In rheumatoid arthritis, synovectomy and debridement are rarely performed and only indicated if all
other measures have failed to control the synovitis. This can be done very effectively arthroscopically.
Articular pannus and cartilage tags are removed at the same time. Meniscal tears are classified
according to their morphology and location in relation to their vascular supply. Red zone tears occur
peripherally in the area supplied by the perimeniscalplexus (10-25%). These tears heal well because
of their adequate blood supply and should be repaired if possible. Am JBJS 2009 v91 p1778-1790.
Cole et al. Surgical management of articular cartilage defects in the knee.

Question 112
With regards to Cubital Tunnel Syndrome what is:
A : Medial intermuscular septum
B : Osbornes fascia
C : Vitamin B6
D : Cubital tunnel
E : Arcade of Struthers
F : Deep flexor pronator aponeurosis
G : Patient education and splinting
H : Steroid Injection
I : NSAIDS
J : Medial epicondyle
1 : The most proximal potential site of compression of the ulnar nerve in the cubital tunnel.
Correct answer:
E
Your answer:
E
2 : The most distal potential site of compression of the ulnar nerve in the cubital tunnel.
Correct answer:
F
Your answer:
A
3 : The most effective nonsurgical treatment of cubital tunnel syndrome.
Correct answer:
G
Your answer:
H

References
Miller ed 5 p438

Question 113

Arthritis in Hand
A : Rheumatoid Arthrits
B : Osteoarthritis
C : Juvenile Chronic Arthrits
D : Psoriatic Arthritis
E : SLE
F : Scleroderma
G : Gout
H : Trauma
I : Muscle contracture
J : Neuromuscular disorder
K : Unknown arthritis
L : Unrelated to arthritis
M : Drug reaction
1 : The nails are often pitted and skin lesions may be evident.
Correct answer:
D
Your answer:
D
2 : Associated with flexion deformity of inter-phalangeal joints, Raynaud's phenomenon and
painful ulcers.
Correct answer:
F
Your answer:
F
3 : Usually affects interphalangeal and first CMC joint.
Correct answer:
B
Your answer:
B

References
Apley's
Eighth

system

of

Orthopaedics

and

Fractures
Edition

Psoriasis of the skin or nails usually preceeds arthritis. The nails are pitted, fingers are deformed due
to erosion and instability of IP joints. The joint changes are similar to those in RA.
Sceleroderma the fingers are smooth skinned and stiff with flexion deformities of IP joints.
Raynauds phenomenon and painful ulcers may develop. Early on, physiotherapy and splinting may
help, later on, digital sympathectomy may be needed to relieve ulcer pain.
Osteoarthritis: The CMC joint is one of the commonest site. This results in adduction of the first MCP
joint. Distal IP joint are also affected.

Question 114
With regards to elbow fracture/dislocations:
A : Ulnar nerve palsy
B : Radial head excision
C : Posterolateral rotatory instability
D : Medial Collateral ligament injury
E : Essex-Lopresti injury
F : Monteggia fracture dislocation
G : Medial epicondyle fracture
H : Posterolateral elbow dislocation

I : Galeazzi fracture
J : Radial head replacement
K : Intraarticular loose body
1 : A 10 year old boy fell from a trampoline. He was brought to the accident and emergency
department with an isolated left elbow injury. On examination the left elbow was swollen and
deformed with some bruising medially. Neurological examination showed loss of finger
extension.
Correct answer:
F
Your answer:
H
2 : A 34 year old lady presented to a wrist surgeon complaining of right wrist pain for three
years. Examination of the right wrist showed tenderness of the distal radioulnar joint. There
was a lateral longitudinal scar at the right elbow and some tenderness at the scar.
Correct answer:
E
Your answer:
E
3 : A 28 year old man presented to the elbow clinic with a history of painful clicking and
intermittent locking of his left elbow. There was history of a sprain of the same elbow 3 years
back. He doesnt feel confident with this elbow especially when doing push ups at the gym.
Correct answer:
C
Your answer:
K

References
Orthoteers, Miller 4th Edition, handbook of fractures by Kenneth Koval 3rd Ed
Monteggia fracture dislocation results usually from a fall on the outstretched hand. Bado classified
them into four types. Posterior interosseous nerve is the commonest nerve palsy associated with all
types of monteggia fractures and this is related to the radial head dislocation since the PIN is closely
approximately to the neck of radius. Essex Lopresti injury is an injury to the interosseous membrane
which can be associated with radial head fracture. The distal radioulnar joint is subluxed due to
proximal migration of radius which causes the wrist pain. This patient has had an excision of the radial
head and at the time of surgery Essex Lopresti injury went unnoticed. Mori described the stages of
elbow dislocation. The lateral ulnar collateral ligament is disrupted initially after a fall on the
outstretched hand. This is followed by disruption of rest of the lateral collateral ligament complex. The
anterior and posterior capsules go next. The anterior band of medial collateral ligament is the last to
rupture.

Question 115
Regarding contracture release of the knee in total knee arthroplasty
A : Common peroneal nerve
B : Popliteus tendon
C : Lateral head of gastocnemius
D : Popliteal artery
E : Iliotibial band
F : Posterior capsule
G : Medial head of gastrocnemius
H : Tibial nerve
I : Lateral intermuscular septum
J : Lateral capsular attachments to tibia
1 : What structure is most commonly released first in a valgus release?
Correct answer:
E

Your answer:
J
2 : In flexion contracture of the knee, after bone resection and removal of osteophytes, which
structure should be released?
Correct answer:
F
Your answer:
F
3 : Which structure is most at risk during lateral release?
Correct answer:
A
Your answer:
A

References
Knee Arthroplasty handbook. Techniques in Total Knee and revision arthropasty. Scuderi G.R, Tria,Jr
A.J,
Springer
2006
The purpose of soft tissue release in the valgus knee is to provide ligamentous balance with
rectangular flexion and extension gaps while maintaining lateral side stability of the knee in flexion.
The structures that can be released in the order most commonly performed are; the iliotibial band,
posterolateral corner, LCL, popliteus, biceps femoris, lateral gastrocnemius, lateral patellar
retinaculum and PCL. In the knee with fixed flexion deformity, following bony resection the first stage
in correction is to remove posterior osteophytes. The next stage involves staged soft tissue release
starting with the posterior capsular elevation, ACL and PCL removal, dissection of posterior capsular
attachments, gastrocnemius attachment release, posterior corner release medially and laterally,
further distal femoral bone resection. The common peroneal nerve is at risk after valgus deformity
correction because of stretching of the nerve leading to a degree of ischaemia. The incidence of CPN
palsy is ~3%

Question 116
Which of the following is the most likely tumour of the proximal humerus in a patient who is :
A : Osteosarcoma
B : Enchondroma
C : Ewing's sarcoma
D : Periosteal chondroma
E : Osteochondroma
F : Chondroblastoma
G : Chondromyxoid fibroma
H : Aneurysmal bone cyst
I : Simple bone cyst
J : Giant Cell Tumor of Bone
1 : A 12 year old with a falling leaf sign on plain radiographs
Correct answer:
I
Your answer:
A
2 : A 12 year old with a lucent epiphyseal lesion surrounded by a distinct sclerotic border on
plain radiographs
Correct answer:
F
Your answer:
C
3 : A 5 year old with a rapidly expanding painful mass
Correct answer:
C

Your answer:
F

References
Core Knowledge in Orthopaedics: Hand, Elbow, and Shoulder Thomas E. Trumble, Roger Cornwall,
Jeffrey E. Budoff, Mosby 2005 RM Wilkins, J Am Acad Orthop Surg, Vol 8, No 4, July/August 2000,
217-224 Miller 4th edition pg 487 and page 466 The "fallen leaf sign" is associated with UBC's and
represents a piece of cortex that has broken off and settled into the fluid filled cavity .In cases of
chondroblastoma, the radiograph demonstrates a lytic lesion with a sharp sclerotic margin and central
stippled or flocculated calcification. A rapidly expanding painful mass in a child could be osteomyelitis
or Ewings sarcoma as both present with very similar features which include fever, elevated WCC and
ESR

Question 117
In septic arthritis of the hip joint, the most common organisms depends on the patient's age
and special circumstances.
A : Staph. aureus
B : Enterobacteriaceae
C : N. gonorrhoeae
D : Brucella
E : Aerobic gram negative bacilli
F : H. influenzae
G : Strep. pneumoniae
H : Mycobacteria
I : Borrelia burgdorferi
J : MRSA
1 : A newborn baby aged 2 months presents with an acute septic arthritis of the hip. The most
common causative organism is:
Correct answer:
A
Your answer:
F
2 : A 26 year old man presents with an acute monoarticular septic arthritis of the knee. The
most common causative organism other than Staph. aureus is:
Correct answer:
C
Your answer:
E
3 : The most common organism in polyarticular septic arthritis other than gonococci is:
Correct answer:
I
Your answer:
A

References
Question 118
Match the most appropriate diagnosis
A : Hyperparathyroidism
B : Hypoparathyroidism
C : Renal osteodystrophy
D : Vitamin D deficiency
E : Hypophosphatasia

F : Osteoporosis
G : Pagets disease
H : Metastasis
1 : Can undergo malignant transformation
Correct answer:
G
Your answer:
G
2 : Decreased alk phos, hypotonia and poor mineralisation of bone
Correct answer:
E
Your answer:
E
3 : Rugger jersey spine on Xray
Correct answer:
C
Your answer:
D

References
Apleys System of Orthopaedics and Fractures, 8th Edition, Solomon, Warwick and Nayagam
Pagets disease has a low risk of undergoing malignant transformation (1%) and usually occurs in the
elderly
and
can
do
so
as
the
first
evidence
of
Pagets
disease.
Hypophosphataemia occurs in a number of disorders in which there in impaired renal tubular
reabsorption of phosphate. Ca2+ levels are normal but there is deficient mineralisation of bone. It can
be familial (X linked genetic dominant inheritance, Vit D resistant, no myopathy). Adult onset
hypophosphataemia rare unexplained bone loss in adults. Responds to phosphate Vit D and
calcium.
Oncogenic
osteomalacia

vascular
tumours,
GCTs,
PVNS.
Renal Osteodystrophy results from chronic renal failure. Its pathology is a variable combination of
rickets or osteomalacia, secondary hyperparathyroidism, osteoporosis and osteosclerosis. Bone
changes are aggravated by aluminium retention or contamination of dialysis fluids. Childrens
radiographs show widened and irregular epiphyseal plates as well as displacement of the
epiphysis( epiphyseolysis). Osteosclerosis is seen in the axial skeleton and may produce a rugger
jersey spine appearance due to alternating bands of increased and decreased bone density. Signs of
secondary hyperparathyroidism may be widespread and severe. Low calcium, high phosphate and
elevated alkaline phosphate predominate.

Question 119
Regarding the pathophysiology of sepsis, which of the above:
A : Kinin
B : Endotoxin
C : IL 8
D : Factor II
E : cGMP
F : IL 6
G : TNF
H : IL 1
I : Prostacyclin
J : Factor VIII
1 : Stimulates T-helper cells?
Correct answer:
H
Your answer:
F
2 : Is a vasodilating substance released from endothelium?

Correct answer:
I
Your answer:
A
3 : Plays a pivotal role via the COX pathway?
Correct answer:
G
Your answer:
I

References
IL 8 and 6 used to down regulate inflammation. Clotting factors are not involved. Endotoxin is a
lipopolysaccharide derived from gram negative bacteria cell wall. IL 1 stimulates T-helper cells to
produce IL2 to stimulate T-cytotoxic cells. Prostacyclin vasodialtes and thromboxane A2
vasoconstricts, both produced by endothelium. TNF plays a pivotal role via COX pathway.

Question 120
What is the most likely diagnosis from the following clinical presentations?
A : Snapping ilio-tibial band
B : Gluteus medius tendonitis
C : Trochanteric bursitis
D : Gilmore's groin
E : Piriformis syndrome
F : Myositis ossificans
G : Labral tear
H : Snapping iliopsoas tendon
I : Hamstring syndrome
J : Sacroiliac dysfunction
1 : Buttock tenderness, pain on sitting and radicular pain in sciatic nerve distribution
Correct answer:
E
Your answer:
J
2 : Audible, deep anterior groin clunk as hip is extended from flexed, abducted and externally
rotated position
Correct answer:
H
Your answer:
A
3 : Painful resisted hip abduction and Trendelenburg gait
Correct answer:
B
Your answer:
G

References
Solheim.
Acta
Segal NA. Arch Phys Med Rehab. 2007

Question 121

Orthop

Scand

1981

Which muscles are the principal cause of the deforming forces which can hinder the reduction
of the following displaced femoral fractures?
A : Gastrocnemius
B : Gluteus Medius
C : Hip Adductors
D : Peroneus Longus
E : Iliopsoas
F : Rectus Femoris
G : Sartorius
H : Short External Rotators
I : Soleus
J : Vastus Medialis
1 : Flexion and exernal rotation of the proximal fragment in proximal 1/3 femoral fractures.
Correct answer:
E
Your answer:
B
2 : Varus angulation of femoral shaft fractures
Correct answer:
C
Your answer:
J
3 : Angulation into recurvatum of supracondylar fractures.
Correct answer:
A
Your answer:
A

References
Campbells
operative
Wheeless' textbook of orthopaedics.

orthopaedics.

10th

ed.

Question 122
In the assessment of a patient with hindfoot pain, which of the following management
strategies would be the most useful initial step in the scenarios?
A : Selective intra-articular injections
B : CRP & Full Blood Count
C : Bone Scan
D : Gait analysis
E : Labelled White cell scan
F : Ultrasound scan
G : CT Scan
H : Examination under Anaesthesia
I : Ankle Arthroscopy
J : Auto-Antibody profile
1 : A 52 year old man with a good history of ankle and subtalar arthritis but radiological
evidence of a well preserved subtalar joint.
Correct answer:
A
Your answer:
G
2 : A 76 year old lady with an early failed total ankle arthroplasty.
Correct answer:
B
Your answer:

B
3 : A 64 year old man with associated clinical findings of tibialis posterior failure and pain
localised to the inferior aspect of the medial malleolus.
Correct answer:
F
Your answer:
F

References
1:
Treatment
Foot

Kelly
of
Ankle

Nunley
acquired
Mar;6(1):167-78.

JA.
flatfoot.

2:
Fox
IM,
Shapero
C,
Kennedy
Tibiotalocalcaneal
arthrodesis
with
intramedullary
interlocking
nail
Clin Podiatr Med Surg. 2000 Jan;17(1):19-31, v.

A.
fixation.

stage
Clin.

IP,
4

adult
2001

Question 123
Match the most appropriate statistical term to the definitions stated below.
A : Positive Predictive value
B : Negative Predictive Value
C : Type 1 (Alpha error)
D : Type 2 (Beta error)
E : Sensitivity
F : Specificity
G : Power analysis
H : Reliability
I : Validity
J : Variance
K : Prevalence
L : Incidence
1 : The ability of a test to detect those patients that have a truly negative test result
Correct answer:
F
Your answer:
B
2 : Is when a null hypothesis is rejected incorrectly (false positive) i.e when a difference was
found by the study but in reality there was no difference.
Correct answer:
C
Your answer:
D
3 : A statistical analysis method of determining the number of subjects needed in a study in
order to have a reasonable chance of showing a difference if one exists.
Correct answer:
G
Your answer:
G
4 : This term refers to the extent to which an experimental value represents a true value
Correct answer:
I
Your answer:
H

References
Statistics section 2 Miller.

Question 124
Which of the answers best anatomically describes the level of amputation
A : Amputation through calcaneo-cuboid and talo-navicular joints
B : Trans distal tibia
C : Through metatarsophalangeal joint
D : Through ankle joint
E : Through mid-shafts of metatarsals
F : Through knee
G : Through navicular-cuneiform and cuboid-metatarsal joints
H : Through tarsal-metatarsal joints
I : Amputation through talo-navicular joint and sub-talar joint
J : Through femur
1 : Chopart's amputation
Correct answer:
A
Your answer:
D
2 : Lisfranc's amputation
Correct answer:
H
Your answer:
C
3 : Syme's amputation
Correct answer:
D
Your answer:
A

References
Chopart amputation: Disarticulation through the midtarsal joint, removing the forefoot and midfoot,
saving talus and calcaneus. Chopart amptutations should not performed for ischemia. It is a very
unstable amputation as most of the tendons which act around the ankle joint have lost their insertion
into
foot
and
the
heel
remains
unstable.
Lisfranc amputation is a partial amputation of the foot at the tarsometatarsal joint, with the sole being
preserved
to
make
the
flap.
Symes amputation includes ankle disarticulation, removal of malleoli, & anchoring heel pad to the wt
bearing surface. It allows excellent gait with a cosmetic prosthesis. Symes amputation will not heal
without a palpable posterior artery pulse.

Question 125
Choose the most appropriate management option for each of the clinical scenarios, all of
which relate to lower limb pathology.
A : Arthroscopic ankle debridement and fusion
B : Medial displacement calcaneal osteotomy
C : Lloyd's procedure
D : Subtalar injection of steroid and anaesthesia
E : Below knee amputation
F : Debridement/removal of hardware and external fixation

G : Osteotomy of the talar dome


H : Pantalar fusion using intramedullary device
I : Non-operative management
J : Above knee amputation
1 : A 62 year old female with rheumatoid arthritis has severe radiographical subtalar and ankle
disease and disabling pain despite analgesia.
Correct answer:
H
Your answer:
A
2 : An 83 year old diabetic patient with absent peripheral pulses and a large infected wound
overlying the lateral aspect of the ankle with radiographical signs of destruction of the ankle
joint
Correct answer:
E
Your answer:
E
3 : A 55 year old male having undergone pantalar fusion for post-traumatic arthritis with
radiographs showing radiolucency around the metalwork and a non-union
Correct answer:
F
Your answer:
F

References
The

Foot

and

Ankle

by

Harold

Kitaoka

and

Deborah

Ravin

Arthroscopic ankle debridement and fusion: Indication, techniques and results. Instr. Course Lect.
1999;
48:
243-8,
10098049
Chou

et

al

(Foot

and

Ankle

International)

2000

Oct;

21

Wheeless's Textbook of Orthopaedics

Question 126
In the following patients, which is the best method of management?
A : NaCl 0.9% IV
B : Dextrose 5% IV
C : Total Parenteral Nutrition via peripheral line
D : Enteral nutrition via nasogastric tube
E : Enteral nutrition via nasojejunal tube
F : Dextrose saline IV
G : Enteral nutrition via percutaneous endoscopic gastrostomy
H : Enteral nutrition via percutaneous endoscopic jejunostomy
I : Total Parenteral Nutrition via central line
J : Hartmann's solution IV
1 : Inability to eat for a minimum of 7-10 days, with a non-functional gut
Correct answer:
I
Your answer:
I
2 : Inability to eat for 2-4 weeks, with a functional gut and a high risk of aspiration
Correct answer:
E
Your answer:

(10):

804-8

E
3 : Inability to eat for 2-3 days, with a functional gut
Correct answer:
F
Your answer:
A

References
First Principles of Gastroenterology: The Basis of Disease and an Approach to Management. Chapter
2, section 7. The NICE guidelines are very prescriptive and informative about this. A non functioning
gut requires TPN A functioning with no aspiration risk requires enteral feeding via the oral route. A
functioning GI tract with risk of aspiration in the short term should have a trial of NG feed with more
invasive steps taken if failure (NJT, PEG, PEJ) etc. Short periods of inadequate intake can be covered
with pre op nutrition supplement and Perioperative IV fluids. SOURCE NICE Guidelines: For
relevance see T&O Competency Based Curriculum for T&O Section 4 2006.

Question 127
What is the term given to the characteristic deformity described?
A : Volkmann's ischaemic contracture
B : Boutonniere's deformity
C : Intrinsic plus deformity
D : Lumbrical plus finger
E : Dupuytren's contracture
F : Intrinsic minus deformity
G : Posterior interosseous nerve palsy
H : Radial nerve palsy
I : Swann neck deformity
J : Proximal median nerve palsy
1 : Flexion at the metacarpophalangeal joints of the fingers with extension of the
interphalangeal joints. The thumb is adducted.
Correct answer:
C
Your answer:
A
2 : Wasting of the small muscles of the hand with moderate clawing. Metacarpophalangeal
joints are extended and interphalangeal joints are partially flexed. The thumb lies flat and
cannot be opposed.
Correct answer:
F
Your answer:
C
3 : Paradoxical extension of distal interphalangeal joint of finger on attempted flexion of joint
following flexor digitorum profundus division at the proximal phalanx.
Correct answer:
D
Your answer:
D

References
Intrinsic Plus Deformity - with the intrinsics tending to flex the MCPs and extend the IPs, when the
intrinsics are tight, the PIP joint cannot flex when the MCP is extended (The MCP extension tightens
up the intrinsics even further so that the PIP cannot be flexed fully). When the MCPs are flexed, the
intrinsics are slackened, and the PIP can flex. This is the basis of the Bunnell test.

Intrinsis Minus Hand: deformity is produced by imbalance of the intrinsic & extrinsics;
intrinsic muscles must be markedly weakened or paralyzed to produce claw deformity; long extensor
muscles hyperextend the MCP joint, & long flexor muscles flex the PIP and DIP joints; PIP joint loses
the ability to extend thru the lateral bands and must rely on the central slip.
Causes:
combined
low
median
and
ulnar
nerve
lesions
brachial
plexus
injuries;
spinal
cord
injuries
Charcot-Marie-Tooth-Disease;
Lumbrical Plus Finger is manifested by intrinsic plus attitude in involved finger on attempted flexion:
( with MCPJ flexion there will be IP extension ); FDP becomes an extensor of the PIP joint; when FDP
relaxes FDS can work with less antagonism and PIP can flex; treatment may involve division of the
lumbrical; Causes: (lumbrical tighter than FDP) - FDP laceration or rupture distal to the Lumbrical
Origin from FDP (the proximal end of the lacerated FDP tendon will retract proximally, drawing the
attached lumbrical proximally as well. The effect is increased tension on the radial lateral band, which
causes the PIP joint to extend); 2) Amputation of the Distal Phalanx (distal to central slip insertion); 3)
Excessively Long Tendon Graft.

Question 128
Select the most appropriate substrate
A : Glucose
B : Amino acids
C : Triglycerides
D : Glucagon
E : Carbohydrate
F : Albumin
G : Nitrogen
H : Skeletal muscle
1 : During trauma what is mobilised as the principle source of energy
Correct answer:
C
Your answer:
H
2 : What is the principle and best energy substrate added to parenteral nutrition if given after
major trauma
Correct answer:
A
Your answer:
B
3 : Depletion of what substrate results in atrophy of the gut mucosa
Correct answer:
B
Your answer:
F

References
Trauma

section

in

Orthoteers

Metabolic
response
to
trauma:
Ebb phase: hypovolaemia and hypoperfusion. Lactic acidosis, inflammatory and catecholamine
response.
Glycogenolysis
persists
as
well
as
lactate
metabolism
Flow phase: Hyperdynamic state. Normalised volume, carbohydrate consumption, amino acid

breakdown to produce carbohydrate. Lipolysis resistant to exogenous sources takes place.


TPN is utilised when the gut is non functional and is higher risk and of higher morbidity than enteral
nutrition. Glucose is utilised as the main carbohydrate source followed by lipid which caries high
calorific value for low volume. Essential and no essential amino acids are added as well as minerals,
vitamins,
trace
elements,
minerals
&
water
Amino acids are essential for gut mucosal health (esp. glutamine) as it is utilised by enterocytes as
their primary energy source ( as well as by the kidney for ammonia excretion. Alanine can be
converted
direct
to
pyruvate
in
the
liver
and
utilised
as
energy.
Source: Essential Surgical Practice, Cuschieri Steele and Moosa, 4th Edition

Question 129
What is the most likely spinal lesion from the list above for the following clinical and
radiological findings?
A : Aneurysmal bone cyst
B : Haemangioma
C : Giant cell tumour
D : Infection
E : Multiple myeloma
F : Osteochondroma
G : Osteoid osteoma
H : Osteosarcoma
I : Eosinophilic granuloma
J : lymphoma
1 : 8 year old child with a painful scoliosis and the pain relieved with aspirin?
Correct answer:
G
Your answer:
G
2 : 40 year old man with back pain. Radiographs show disc space narrowing and end plate
erosions.
Correct answer:
D
Your answer:
D
3 : 8 year old child with progressive thoracic back pain. Lateral radiographs show vertebral
flattening.
Correct answer:
I
Your answer:
I

References
Millar
4th
Apley's system of Orthopaedics and Fractures. 8th Edition

Question 130
Imaging in orthopaedic practice
A : Plain radiograph
B : CT
C : MRI
D : MRI with gadolinium
E : Ultrasound scan
F : Arthrogram

Edition

G : Indium scan
H : Bone scan
I : Myelogram
J : EMG
1 : Which imaging modality has the highest sensitivity and specificity for the early detection of
osteonecrosis?
Correct answer:
C
Your answer:
D
2 : The procedure of choice for diagnosing a subarachnoid cyst.
Correct answer:
C
Your answer:
B
3 : Recurrent disc herniation is best diagnosed with:
Correct answer:
D
Your answer:
D

References
Question 131
A 35 year old taxi driver involved in a road traffic accident. He sustained a posterior
dislocation of his right hip. Choose the most appropriate option for further management
A : Plain Xray
B : CT Scan
C : MRI Scan
D : Nerve Exploration
E : Nerve Conduction studies
F : Ankle Foot Orthosis
G : Urgent Open reduction
H : Emergency Open reduction
I : Fluoroscopic Stress Test
1 : 35 year old taxi driver involved in a road traffic accident. He sustained a posterior
dislocation of his right hip. This was reduced in theatre under GA. Check x-ray post reduction
shows an incongruent hip joint
Correct answer:
B
Your answer:
B
2 : This was reduced in theatre under GA. New onset sciatic nerve palsy is noted in the
recovery room
Correct answer:
H
Your answer:
A
3 : This was reduced satisfactorily. 3 months post reduction complains of increasing pain in
right hip
Correct answer:
C
Your answer:
I

References
Rookwood
and
Green's
Fracture
in
Adults
Vol2
pg
1561-1566
Trauma.
Court
Brown
Lippincott
Williams
&
Wilkins
2006
p
259-264
Emergency closed reduction of hip dislocation is mandatory unless the patients overall condition
precludes this. If following reduction, the hip joint is found to be incongruent at postoperative AP and
lateral plain x-ray, a CT can be performed to look for intraarticular bone fragments. If the hip is
irreducible, secondary to interposition of soft tissue, femoral neck fracture or buttonholing through the
posterior capsule, open reduction is indicated to preserve the blood supply to the head. Other
indications for open reduction include sciatic nerve palsy post reduction and associated femoral neck
fracture. Complications of posterior hip dislocation include, avascular necrosis (20%), osteoarthritis
(25%), heterotrophic ossification (5%) and sciatic nerve damage (10%). MRI Scan remains the gold
standard technique for assessment of avascular necrosis in the hip

Question 132
Regarding the management of suspected infection in a THR
A : CRP
B : Technetium (99) isotope bone scan
C : Plain radiograph of hip
D : MRI of hip
E : ESR
F : WCC
G : Multiple intra-operative tissue cultures
H : Blood cultures
I : Wound swab
J : Image guided joint aspiration
1 : Which blood test is the best indicator of deep infection in a THR in the 1st year following
surgery?
Correct answer:
A
Your answer:
A
2 : Which investigation has a low sensitivity but high specificity for infection?
Correct answer:
J
Your answer:
F
3 : Which method is regarded as the gold standard in the diagnosis of an infected THR?
Correct answer:
G
Your answer:
J

References
Shilh LY et al. ESR and CRP values in patients with THA. Clin Orthop 1987;225:238-46.
Reing CM et al. Differential bone scanning in the evaluation of a painful total joint replacement.
JBJS(Am)1979
61(A):933-6
Spangehl MJ et al. Prospective analysis of pre-operative and intra-operative investigations for the
diagnosis of infection at the sites of 202 revision total hip arthroplasties. JBJS 1999. 81(A)672-83

Question 133

The following gait abnormalities can be caused by specific problems with a prosthesis. Match
the most likely prosthesis problem with the gait abnormality
A : Short prosthesis
B : Long prosthesis
C : Poor socket fit
D : Painful stump
E : Knee too anterior
F : Excessive knee rotation
G : Quadriceps weakness
H : Heel too soft
I : Heel too hard
J : Weak abductors
1 : Short stance phase
Correct answer:
D
Your answer:
D
2 : Circumducted gait
Correct answer:
B
Your answer:
G
3 : Lateral trunk bending
Correct answer:
A
Your answer:
B

References
Miller. Review of Orthopaedics. Rehabilitation: Gait, amputations, prosthetics, Orthotics

Question 134
Match the correct normal values for the following angles measured in a normal knee Xray:
A : 3 degrees varus
B : 70 - 75 degrees
C : 0 degrees
D : 91 - 100 degrees
E : 80 - 90 degrees
F : 77 - 84 degrees
G : 124 -136 degrees
H : 165 -175 degrees
1 : What is the normal aMPTA (anatomic medial proximal tibial angle) in AP view?
Correct answer:
A
Your answer:
F
2 : What is the normal aLDFA (anatomic lateral distal femoral angle) in AP view?
Correct answer:
D
Your answer:
F
3 : What is the normal aPTTA (anatomic posterior proximal tibial angle) or posterior tibial slope
in lateral view?
Correct answer:
F

Your answer:
F

References
Paley
Principles of deformity correction

Question 135
With regards to fractures of the distal radius and percutaneous k wire fixation.
A : Extensor carpi radialis longus
B : Posterior interosseous nerve
C : Brachioradialis
D : Radial nerve
E : Flexor pollicis longus
F : Extensor pollicis longus
G : Superficial radial nerve
H : Recurrent motor branch of the median nerve
I : Median nerve
1 : 6 weeks post MUA & k wiring a patient presents with sensory disturbance in the first web
space. The affected nerve travels under the cover of this muscle.
Correct answer:
C
Your answer:
A
2 : 6 weeks post MUA & k wiring a patient presents with reduced range of active thumb
movement due to damage to this structure.
Correct answer:
F
Your answer:
H
3 : Cotton-Loder position is associated with compression of which nerve?
Correct answer:
I
Your answer:
I

References
Fractures
of
the
distal
radius
(Green's
operative
hand
surgery,
5th
Ed)
Lasts
anatomy
10th
Ed
The superficial branch of radial nerve is commonly injured with percutaneous k-wiring technique. This
can result in painful paresthesias in the first web space. The superficial radial nerve travels in the
forearm on the radial aspect of radial artery under the brachioradialis muscle. Extensor pollicis longus
can be injured with MUA & K-wiring. This results in loss of retropulsion which is primarily caused by
EPL. Cotton-loder position is a position of extreme of flexion and ulnar deviation which has been
shown to cause a reduction in carpal tunnel volume leading to compression of median nerve.

Question 136
Regarding flexor tendon injuries of the hand:
A:1
B:2
C:3

D:4
E:5
F:6
G:7
H:8
I:9
J : 10
1 : Which flexor tendon zone contains the A2 pulley
Correct answer:
B
Your answer:
B
2 : How many phases occur in tendon healing?
Correct answer:
C
Your answer:
D
3 : How many months does it take before full tensile strength is recovered following flexor
tendon repair on a strength-duration curve?
Correct answer:
H
Your answer:
J

References
Green's

Operative

Hand

Surgery

5th

Edition

A.
Zone
1
is
distal
and
Zone
5
is
proximal.
The
five
zones
are
1 - contains flexor digitorum profundus only distal to the insertion of flexor digitorum superficialis
2 - from insertion of flexor digitorum superficialis to the proximal edge of the A1 pulley
3 - from the proximal edge if the A1 pulley to the distal edge of the carpal tunnel

4
within
the
carpal
tunnel

5
proximal
to
the
carpal
tunnel
B.
There
are
3
phases
of
tendon
healing:
The initial inflammatory phase, which lasts about 24 hours, erythrocytes, platelets and inflammatory
cells (eg: neutrophils, monocytes and macrophages) migrate to the wound site and clean the site of
necrotic materials by phagocytosis. In the meantime, these cells release vaso active and chemo tactic
factors which recruit tendon fibroblast to begin collegan synthesis and deposition.
A few days after the injury, the repairing phase begins. In this phase, which lasts a few weeks, tendon
fibroblast synthesise abundant collegan and other extra cellular matrix components such as
proteoglycans
and
deposit
them
at
the
wound
site.
After about 6 weeks, the remodelling phase starts. This phase is characterised by decreased
cellularity and decreased collagen and glycosaminoglycan synthesis. During this period, the repair
tissue changes to fibrous tissue, this again changes to scar like tendon tissue after 10 weeks.
C.
Effects
of
Time
on
Tendon
Healing:
- strength duration curve show that healing is weak at 21 days, but of sufficient strength to tolerate
active
contraction
of
muscle;
- at 6 weeks, external elastic traction can be applied if force is not excessive;
some
surgeons
allow
gentle
active
ROM
at
6
weeks;
- at 3 months, moderate stress can be applied to the flexor tendon in both flexion and extension;
- at 8 months, full tensile strength has been recovered

Question 137
Which is the single most likely diagnosis in each of the following scenarios?

A : C8/T1 root avulsion


B : Medial cord injury
C : High ulnar nerve injury
D : Low ulnar nerve injury
E : Radial nerve injury at elbow
F : Posterior cord injury
G : Lateral cord injury
H : High median nerve injury
I : Low median nerve injury (at wrist)
J : Axillary nerve lesion
1 : Loss of power of elbow flexion
Correct answer:
G
Your answer:
I
2 : Loss of thumb IPJ flexion but preservation of FDS power to ring and little fingers
Correct answer:
H
Your answer:
H
3 : Weakness of all hand intrinsic muscles with ipsilateral Horners syndrome
Correct answer:
A
Your answer:
A

References
Question 138
For each of the clinical scenarios listed, select the most appropriate management:
A : Early active motion without splintage
B : Cast immobilisation for 4 weeks
C : Closed reduction and casting
D : Open reduction and internal fixation with 2 or more screws alone
E : Open reduction and internal fixation with plate and screws
F : Transverse k-wire fixation
G : Longtitudinal k-wire fixation
H : Wound management and splintage
I : External fixation
J : Buddy strapping for comfort, with early movement
K : Further information required
1 : A spiral fracture of the second metacarpal shaft, greater than 2 shaft diameters in length,
with rotation and shortening
Correct answer:
D
Your answer:
E
2 : A short spiral fracture of the second metacarpal shaft, less than 2 shaft diameters in length,
with rotation and shortening
Correct answer:
E
Your answer:
E
3 : An intraarticular fracture of the base of the fifth metacarpal, undisplaced on AP view
Correct answer:
K

Your answer:
B

References
http://www.wheelessonline.com/ortho/hand_and_metacarpal_fractures accesed 2 Dec 09

Question 139
Which is the most likely complication for the described scenarios?
A : Acetabular wear
B : Aseptic loosening of the femoral stem
C : Avascular necrosis of the femoral head
D : Deep infection
E : Femoral neck non-union
F : Increased risk of anterior dislocation
G : Increased risk of posterior dislocation
H : Peri-prosthetic fracture
I : Trendelenberg gait
1 : A 55 year old lady involved in a RTA sustains a displaced intra-capsular femoral neck
fracture. She undergoes closed reduction and percutaneous cannulated screw fixation. She is
complaining of groin pain four months post op.
Correct answer:
C
Your answer:
C
2 : A 30 year old man has a hip resurfacing procedure for long standing AVN of the femoral
head. Pre-op he was found to have a small head/neck ratio. Post-operatively he has severe
pain and inability to bear weight.
Correct answer:
H
Your answer:
H
3 : Injury to superior gluteal nerve.
Correct answer:
I
Your answer:
I
4 : A 45 year old man with a long history of alcohol abuse had a trip and fall and sustained
displaced fracture of his femoral neck. He has a cemented total hip replacement through a
posterior approach with satisfactory restoration of femoral offset and neutral version.
Correct answer:
G
Your answer:
H

References
Rockwood and Green's Fractures in Adults: Robert W. Bucholz, James D. Heckman, Charles M.
Court-Brown,
Paul
Tornetta,
Kenneth
J.
Koval
Wheeless
online
Orthoteers
online
Importance of awareness of common complications and mechanisms by which they may happen,
thus avoiding them in the future

Question 140
With regards to surgery for syndactyly of the hand:
A : Thumb-index
B : Before 6 months
C : Scar contracture/deformity
D : Ring-small
E : Before 18 months
F : Circulatory insufficiency
G : Index-middle
H : Distal web migration
I : Middle-ring
J : After 18 months
1 : What is the most common ray for syndactyly?
Correct answer:
I
Your answer:
I
2 : What is the most common occurring complication following surgery for syndactyly?
Correct answer:
C
Your answer:
F
3 : With regards to syndactyly between rays of unequal length, when is it best to perform
corrective surgery?
Correct answer:
B
Your answer:
E

References
Campbells

Operative

Orthopaedics

10th

Edition

Miller Review of Orthopaedics 4th Edition

Question 141
A 50 year-old woman complains of acute pain in her right foot and increase wear on the medial
border of her shoes. On examination she has pes planus and the classic too many toes sign
on the right foot .
A : Tibialis posterior
B : Peroneus brevis
C : Tibialis anterior
D : Peroneus longus
E : Extensor hallucis longus
F : Flexor digitorum longus
G : Peroneus tertius
H : Extensor digitorum longus
1 : Which is the most likely tendon to be dysfunctional?
Correct answer:
A
Your answer:
A
2 : The patient underwent surgery to correct her symptoms successfully. Which tendon
transfer is most likely to have been performed?

Correct answer:
F
Your answer:
G
3 : Which tendon is the true antagonist of the dysfunctional tendon?
Correct answer:
B
Your answer:
C

References
Apleys System of Orthopaedics and Fractures. Solomon, Hodder Arnold 2001
Rupture of tibialis posterior tendon is not always obvious, it should be carefully tested for especially in
those cases where the deformity develops rapidly in one foot. Classically on examination, the foot
appears pronated, the arch is flat and the navicular looks prominent. The heel is in valgus and from
behind too many toes are seen lateral to the outer edge of the heel. Acute flat foot - think tibialis
posterior rupture. In young and physically active patients, operative repair or tendon transfer with
flexor digitorum longus is worthwhile

Question 142
Autonomous dermatomal zones
A : C5
B : C6
C : C7
D : C8
E : L4
F : L5
G : S1
H : L2
I : L3
J : None
1 : Tip of the middle finger
Correct answer:
C
Your answer:
C
2 : Skin over the medial malleolus
Correct answer:
E
Your answer:
E
3 : Lateral Cutaneous nerve of the thigh
Correct answer:
J
Your answer:
I

References
Last's Anatomy

Question 143

Of the following scapular injuries which is the correct answer to the following questions?
A : Glenoid neck fracture
B : Glenoid fossa fracture
C : Scapular body fracture
D : Acromium fracture
E : Coracoid fracture
F : Scapulothoracic dissociation
G : Avulsion fracture of the scapula
H : Ipsilateral fracture of the clavicle and glenoid neck
1 : Is classified according to Eyres
Correct answer:
H
Your answer:
A
2 : Is classified according to Ideberg?
Correct answer:
B
Your answer:
C
3 : Is associated with injury to the brachial plexus and subclavian/axillary arteries?
Correct answer:
F
Your answer:
H
4 : Is the commonest fracture?
Correct answer:
C
Your answer:
A

References
Review
of
Orthopaedic
Rockwood
and
Greens:
Fractures
Rockwood and Matsen The Shoulder 2nd ed

Trauma.
in

Adults

Brinker
6th

2001
Edition

Question 144
With regards the ligaments of the knee?
A : Lateral tibial plateau fracture
B : Medial tibial plateau fracture
C : Anterior cruciate ligament
D : Posterior cruciate ligament
E : Medial collateral ligament
F : Lateral collateral ligament
G : Popliteus tendon
H : Ilio-tibial band
I : Popliteo-fibular ligament
J : Biceps femoris tendon
1 : This ligament primary role is controlling rotation of the knee?
Correct answer:
C
Your answer:
I
2 : This ligament forms the main structural component of the postero-lateral ligament
complex?
Correct answer:

I
Your answer:
J
3 : A Segond fracture is associated with rupture of the anterior cruciate ligament and which
other structure?
Correct answer:
A
Your answer:
F

References
Question 145
The most likely diagnosis
A : Primary Hypertension
B : Gastric Ulcer
C : Osteoporosis
D : Secondary Hypertension
E : Diabetes Mellitus
F : Pathological Fracture
G : Wound Infection
H : Hypoadrenalism
I : Septic Arthritis
J : Proximal myopathy
1 : Middle-aged lady taking oral corticosteroids for 20 years develops severe lower back pain.
No history of trauma.
Correct answer:
F
Your answer:
F
2 : 40 year old man taking prednisolone develops gastro-enteritis. 5 days later he is brought by
ambulance to the Accident and Emergency department with a temperature of 38.0 degrees
centigrade, abdominal pain, weakness, confusion, and severe postural hypotension.
Correct answer:
H
Your answer:
H
3 : 50 year old lady with features of right knee osteoarthritis is given an intra-articular injection
of depomedrone. 1 Day later she presents to Accident and Emergency with severe knee pain,
pyrexia of 38.0 degrees, and general malaise.
Correct answer:
I
Your answer:
I

References
Question 146
Regarding Dupuytren's disease:
A : Grayson's ligament
B : Cleland's ligament
C : Lateral digital sheet
D : Natatory ligament
E : Pretendinous band

F : Spiral band
G : Natatory cord
H : Central cord
I : Spiral cord
J : Lateral cord
1 : Part of normal digital fascia that does not become involved with Dupuytren's disease
Correct answer:
B
Your answer:
C
2 : Involvement of this portion of the palmar fascia will cause contracture of the
metacarpophalangeal joint
Correct answer:
E
Your answer:
D
3 : This structure forces the neurovascular bundle in a palmar direction and towards the
midline as the disease progresses.
Correct answer:
I
Your answer:
G

References
Campbell,
10th
ed.
Thomine describes a longitudinally oriented fascia located dorsal to the neurovascular bundle, which
he terms the retrovascular cord. This structure often is involved in the disease and may be implicated
as a cause for recurrent PIP contractures. The Cleland ligament generally is believed to be spared.
The pretendinous cord nearly always is responsible for primary contracture of the
metacarpophalangeal joint. It may attach to the distal palmar crease skin, base of the proximal
phalanx, or the tendon sheath at this level, or it may extend to attach to the flexor tendon sheath over
the middle phalanx or the skin in this area. A spiral cord occurs when four normally existing structures
(pretendinous band, spiral band, lateral digital sheet, and the Grayson ligament) become diseased.
The spiral cord runs dorsal to the neurovascular bundle proximally and volar to it distally. When the
spiral cord is contracted, the neurovascular bundle is drawn toward the midline of the finger.
Neurovascular displacement is found most commonly on the ulnar aspect of the little and ring fingers,
and tedious dissection is required to prevent digital nerve injury.

Question 147
With regards to calcium absorption from gut
A : Glucocorticoids
B : Gluten enteropathy
C : Excess thyroid hormones
D : Phosphate depletion
E : Metabolic acidosis
F : Phenytoin
G : Aluminium containing antacids
H : Rifampicin
I : Cholestyramine
J : Phonobarbital
1 : Increases gastrointestinal calcium absorption
Correct answer:
D
Your answer:
G

2 : Decreases serum calcium by raising the ratio of ionised to unionised calcium


Correct answer:
E
Your answer:
F
3 : Inhibits intestinal phosphate absorption by binding to phosphate in the intestine
Correct answer:
G
Your answer:
F

References
Favus MJ, ed:primer on the metabolic bone diseases, 3rd ed. Philadelphia, lippincort-Raven,1996
CommentsCalcium homeostasis is regulated by PTH therefore an excess of thyroid hormones can
lead to an increase in calcium absorption. Half of calcium is protein bound the other half is either
ionised (most) or bound to HCO3 citrate etc. Free ionised calcium is essential for muscle function,
nerve conduction and coagulation. Some calcium is absorbed in the gut but this is minor (2.5mmol) in
comparison with the kidney (247.5mmol. The kidney filters large amounts and 99% is reabsorbed
(60% in PCT and the rest in ascending limb of loop of Henle. DCT reabsorption is regulated by
parathormone. Phosphate is essential for the Calcium ATP cycle. Most is reabsorbed in the PCT
initiated by parathormone. It is also absorbed in the upper GI tract passively. Many stimuli that
increase Calcium absorption also increase Pi absorption however PTH is phosphaturic If phosphate is
depleted then less calcium will be complexed to phosphate meaning a higher level of ionised calcium,
in addition phosphate depletion is triggered by PTH, which also raises levels of calcium. This will
reduce serum concentration through negative feedback via the parathyroid glands. Metabolic acidosis
leads to a drop in pH, this means that there are more bicarbonate ions available to bind to ionised
calcium however this represents only a small proportion of diffusible calcium (about 1/8). Protein
bound calcium increases as pH rises as the plasma proteins become more ionised therefore binding
calcium, therefore serum calcium decreases Aluminium containing antacids are contraindicated in
hypophosphataemia. This is due to the fact that aluminium binds to form an insoluble acid with
phosphate is therefore not absorbed from the gut and reduces phosphate absorption. This is not an
issue in normal healthy subjects. Phenytoin binds calcium in the gut therefore additional calcium
supplementation is recommended when it is prescribed in those with immobility, inadequate sun
exposure or dietary insufficiency. Source: Ganong Physiology 17th Edition. Lange BNF

You might also like